You are on page 1of 48

Seorang perempuan 55 tahun MRS dengan sesak napas dan lelah.

Tekanan darah 110/90, HR 110 kali/menit,


Frekuensi pernapasan 30 kali/menit. Pada CXR terlihat kardiomegali dan tanda-tanda edema intertisial. Pada
ekokardiografi, ukuran ventrikel kiri besar dari besar dan ketebalan dinding tetap, hanya terlihat gerakan
hipokinetik menyeluruh.
Diagnosis kasus diatas adalah …
a. Ischemic cardiomyopathy
b. Cor pulmanale chronicum
c. Congestive cardiomyopathy
d. Asymmetrica septal hyperthrophy
e. Idiopathic hyperthropy subaorta stenosis
6. Seorang perempuan usia paruh baya dengan hiperkolesterolemia MRS dengan nyeri dada hebat menjalar ke
lengan kiri. Ada serangan jantung. Maka obat yang diberikan adalah …
a. Infus thrombin
b. Infus fibrinogen
c. Infus thrombosit
d. Transfuse trombosit
e. Infus Tissue Plasminogen Activator
7. Pasien 63 tahun pernah mendapat serangan IMA 9 bulan yang lalu. Saat ini pasien sehat dan bugar,
meminta izin kepada dokter untuk mengikuti fun-aerobik di halaman PKM. Dokter memeriksa TTV dan
melarang pasien untuk mengikuti latihan.

Syarat HR max adalah …


a. 60-70% dari 157
b. 70-85% dari 157
c. 80-85% dari 157
d. 85-90% dari 157
e. 90-95% dari 157
8. Pasien 63 tahun pernah mendapat serangan IMA 9 bulan yang lalu. Saat ini pasien sehat dan bugar,
meminta izin kepada dokter untuk mengikuti fun-aerobik di halaman PKM. Dokter memeriksa TTV dan
melarang pasien untuk mengikuti latihan.

Syarat HR max adalah …


a. 60-70% dari 157
b. 70-85% dari 157
c. 80-85% dari 157
1
d. 85-90% dari 157
e. 90-95% dari 157

10. Disosiasi elektromekanik pada gambaran EKG dapat disebabkan oleh kelainan.
Kelainan di bawah ini yang dapat menyebabkan hal di atas, kecuali …
a. Kegagalan miokard
b. Rupture dinding jantung
c. Emboli paru
d. Temponade jantung
e. Pericarditis
11. Perempuan 30 tahun datang dengan keluhan jantung berdebar-debar. Pada pemeriksaan fisi didapatkan BP
120/80 mmHg, HR 180x/menit, Frekuensi Pernapasan 28x/menit, dan temperature 36,8 derajat Celcius.
Apakah tindakan yang dilakukan sebagai pertolongan awal?
a. Defibrilasi
b. Kardioversi
c. Cardiac thump
d. RJP
e. Manuver vasovagal
12. Seorang perempuan berusia 20 tahun datang ke RS dengan keluhan jantung berdebar-debar dan rasa mau
terjatuh sejak beberapa hari terakhir. Gejala ini tidak pernah terjadi sebelumnya. Selama ini pasien tidak
pernah minum obat dan tidak pernah sakit. Pada pemeriksaan fisi didapatkan: TD 110/80 mmHg, DN
180x/menit dan FP 24x/menit. Diagnosis yang didapatkan pada pasien ini adalah aritmia jantung.

Apakah jeis aritmia yang paling mungkin terjadi?


a. Atrial Fibrilasi
b. 3rd degree AV Block
c. Ventrikular Takikardi
d. Supraventricular Takikardi

13. Seorang perempuan berusia 35 tahun datag ke IGD dengan keluhan jantung berdebar-debar yang dialami
sejam yang lalu. Pasien selama ini mengkonsumsi obat hipertensi dan angina. Pada pemeriksaan fisi
didapatkan TD 160/100 mmHg, DN 120x/menit, FP 24x/menit, dan suhu badan 37°C.
Apakah jenis obat yang menyebabkan keluhan pada pasien?
a. Captopril
b. Nifedipine
c. Diltiazem
2
d. Verapamil
e. Propranolol

Seorang laki-laki berusia 55 tahun datang ke IGD RS dengan keluhan utama sesak napas yang dialami 24 jam
yang lalu disertai dengan rasa berdebar-debar. Pasien juga merasa sedikit pusing dan berkeringat dingin.
Tidak ada riawayat infark sebelumnya, ada riwayat hipertensi lama dan riwayat merokok sigaret. Pada
pemeriksaan fisis: TD 80/50 mmHg, DN 186x/menit teratur, dan FP 26x/menit. Pada auskultasi terdengar
adanya krepitasi bilateral basal paru, DVJ R+5 pada posisi 30°, terdengar bising holosistolik derajat III/VI di
daerah apex jantung yang menjalar ke axilla.
Apakah diagnosis yang paling mungkin?
a. Regurgitasi mitral dengan takikardi ventrikel
b. Kardiomiopati kongestif dengan fibrilasi atrium
c. Stenosis aorta dengan takikardi supraventricular
d. Regurgitasi mitral dengan takikardi supraventricular
e. Regurgitasi aoirta dengan fibrilasi atrium

16. Pasien dengan diagnosis gagal jantung kronik.


Obat gagal jantung yang dapat menyebabkan sinus bradikardi, interval PR memanjang, dan ekstrasistole
ventrikel adalah …
a. HCT (Hydrochlorothiazide)
b. Kaptopril
c. Furosemide
d. Digoxin
e. Isosorbide Dinitrat
17. Senyawa di bawah ini untuk memecah thrombus adalah …
a. Antithrombin III
b. Thromboplastin
c. Thrombin
d. Plasmin
e. Thrombomedulin
18. Seorang pria berumur 35 tahun datang ke praktek dokter spesialis dengan nyeri dada kiri yang menjalar ke
lengan kanan. Timbul baru pertama kali, nyeri 1-2 menit, tidak ada riwayat trauma dada. EKG menunjukkan
elevasi segmen ST di sadapan V1-V4. Diberi nitrat sublingual dan 5 menit kemudian nyeri hilang. Dilakukan
EKG ulang dan hasil normal. Pemeriksaan Lab menunjukkan CKMB 10 U/L dan Troponin T negative.

Diagnosis kasus di atas adalah ...


3
a. Prinzmetal angina
b. Angina stable
c. Physiology reaction myocardial infarct
d. Infark miokard subendokardial
e. Infark miokard anterior lama

21. Seorang laki-laki 43 tahun datang ke RS dengan keluhan sesak disertai nyeri dada, batuk, dan demam juga
dikeluhkan. Pada pemeriksaan fisis ditemukan TD 180/100 mmHg, pulsus paradoxus, DN 112x/menit, JVP
R+3. Pada EKG didapatkan “low QRS voltage”. Pasien ini diduga menderita efusi perikard dan hasil CT-Scan
menyatakan adanya efusi perikard.
Dimanakah letak ruang yang mengalami efusi tersebut?
a. Tunica adventisia
b. Subendokardium
c. Endocardium
d. Myocardium
e. Epicardium

31. Seorang laki-laki 65 tahun dengan infark miokard akut anterior berat beberapa hari lalu. Ditemukan
pericardial friction rub dan nyeri dada pleuritik yang sudah sulit dikontrol dengan narcotic ataupun steroid.
Tiba-tiba pasien mengalami hipotensi. Tidak ada murmur ditemukan.
Etiologi yang paling mungkin pada keadaan akut di atas adalah …
a. Regurgitasi mitral akut akibat rupture m. papillaris
b. Rupture septum ventrikel
c. Ruptur dinding jantung external/posterior
d. Infark ventrikel kanan
e. Infark miokard yang luas

32. Seorang penderita laki-laki 65 tahun dengan riwayat hipertensi, DM datang ke UGD dengan keluhan nyeri
dada yang timbul tiba-tiba disertai rasa sesak napas. Pemeriksaan fisis menunjukkan adanya kelainan
pernapasan, kecemasan disertai keringat dingin. Tekanan darah 180/100 mmHg, DJ 170x/menit dengan irama
irregular, RR 40x/menit, terdengar ronki basah halus 2/3 bagian dari basal kedua paru, desakan vena jugularis
12 cm, terdengar S3 dan bising holosistolik derajat 2/6 di apeks kordis. Foto thorax menunjukkan adaya
pembesaran jantung dan gambaran edema paru.
Diagnosis yang paling mungkin adalah …
a. Acute anterior myocardial infarction
b. Acute inferior myocardial infarction
c. Right ventricular infarction
d. Hypertensive heart failure
4
e. Cor pulmonale chronicum

33. Seorang laki-laki 28 tahun mengunjungi klinik untuk pemeriksaan fisis rutin. Dia tidak mengeluhkan
sesuatu, tidak ada riwayat pingsan dan tidak ada riwayat keluarga yang menderita penyakit jantung. Pada
pemeriksaan fisis terdengar adanya bising sistolik derajat II/VI yang terdengar jelas di sela iga kanan bawah.
Bising jantungnya meningkat intensitasnya bila penderita menarik napas dan menurun apabila dilakukan
maneuver valsalva.
Penyebab dari bising yang ditemukan pada penderita ini adalah …
a. Flow murmur
b. Aortic regurgitation
c. Mitral regurgitation
d. Tricuspid regurgitation
e. Hyperthropic cardiomyopathy

35. Seorang laki-laki 35 tahun mengalami cedera pada punggung sebelah kanan setelah kecelakaan. Penderita
pergi ke dukun dan dilakukan pemijatan pada daerah punggung dan perut. Setelah 3 hari penderita mengeluh
kaki kanan bengkak, makin membesar dan timbul rasa nyeri. Kulit kaki terlihat agak kecoklatan.
Kasus di atas mengarah pada …
a. Varises vein
b. Diabetic foot
c. Fraktur kaki terbuka
d. Acute limb ischemic
e. Deep vein thrombosis
36. Seorang pendaki gunung tiba-tiba jatuh dan mengalami patah tulang panggul kanan. Beberapa jam
kemudian penderita mengeluh kaki kanannya sakit dan jari-jari kaki sulit digerakkan. Kulit kaki kanan
berubah warna dari kemerahan menjadi biru kehitaman.

Penderita mengalami suatu kondisi yang disebut …


a. Ischialgia
b. Acute limb ischemia
c. Chronic limb ischemia
d. Deep vein thrombosis (DVT)
e. Chronic Critical Limb Ischemia
37. Lapisan endocardium dari dalam ke luar adalah …
a. Lapisan endokardium, miokardium, epikardium
b. Lapisan endocardium, miokardium, pericardium
5
c. Lapisan endothelial, subendotelial, endocardium, elastikomuscular
d. Lapisan endotel, subendotel, elastikomuskular, subendokardium
e. Lapisan endotel, subendotel, subendokardium, elastikomuskular
38. Ciri khas arteri besar adalah
a. Terdapat serabut kolagen II dan IV pada lapisan tunika media
b. Terdapat banyak serat elastis pada tunica media
c. Terdapat tunika elastika interna
d. Terdapat otot polos pada tunika adventitia

40. Seorang laki-laki 48 tahun mengeluhkan sesak saat beraktivitas disertai nyeri dada. Hasil pemeriksaan
ekokardiografi menunjukkan orificum katup aorta <0,70 cm².
Tindakan yang paling cocok untuk penderita ini adalah …
a. Angiografi coroner
b. Konsul ahli bedah thorax
c. Evaluasi ekokardiografi 6 bulan kemudian
d. Uji latih jantung untuk menyingkirkan adanya PJK
e. Evaluasi foto thorax dan ekokardiografi 2 tahun kemudian
41. Seorang laki-laki 50 tahun datang ke poliklinik dengan keluhan nyeri pada tungkai kanan ketika berjalan.
TD 150/90 mmHg, HR 110x/menit, riwayat merokok 1 bungkus/hari.

Pemeriksaan yang dianjurkan adalah …


a. Melakukan pemeriksaan 6 P
b. Melakukan pemeriksaan Doppler USG kaki
c. Melakukan ekokardiogram
d. Melakukan pemeriksaan kekentalan darah
e. Melakukan pemeriksaan EKG

43. Fatty streak dibentuk oleh …

a. Kolesterol HDL
b. Kolesterol LDL
c. Trigliserida
d. Apolipoprotein A
e. LDL teroksidasi

6
44. Seorang pria 60 tahun dengan Hipertensi dan riwayat penyakit jantung CABG 2 tahun lalu. Rehabilitas
pasien diatas termasuk dalam …
a. Fase 1
b. Fase 2
c. Fase 3
d. Fase 4
e. Fase kronik
45. Kultur yang dapat membantu untuk mendiagnosis IE adalah …
a. Kultur darah pada blood agar
b. Kultur darah pada sabaroud dextro agar
c. Kultur darah pada Theyer-Martin agar
d. Kultur darah pada Lowenstein Jensen
e. Kultur pada medium EMJH
46. Laki-laki 48 tahun IMT 30 kg/m2 riwayat pernah menjalani Precutaneus coronary intervension (PCI)
selama 6 bulan lalu, hasil pemeriksaan LDL 129 mg %.
Berapa nilai LDL yang disarankan pada pasien di atas?
a. <100%mg
b. <120%mg
c. <130%mg
d. <140%mg
e. <160%mg
47. Seorang pria berusia 45 tahun datang ke RS melakukan perawatan rutin yang diketahui menderita
gangguan profil lipid. Diketahui LDL 230 mg/dl, trigliserida 280 mg/dl, HDL 30 mg/dl. Penderita telah
menerapkan diet dan berolahraga, tetapi hal tersebut tidak mempengaruhi profil lipidnya, sehingga ia
mengonsumsi obat penurun profil lipid yakni simvastatin. Sebelum menerapkan terapi pasien harus
memeriksakan hatinya dan dilakukan secara berulang tiap 3 bulan. Selain simvastatin obat apalagi yang dapat
menurunkan kadar profil lipid pasien…
a. Ezitimibe
b. Fenilfibrat
c. Gemfibrozil
d. Asam nikotinat
e. Bile-sequestring resin

7
50. Yang tidak memiliki lamina elastika interna tetapi memiliki lamina elastika externa adalah …
a. Arteri sedang
b. Arteri besar
c. Arteri kecil
d. Kapiler
e. Sinusoid

53. Wanita usia 15 tahun datang dengan keluhan nyeri dan bengkak pada sendi kaki disertai demam, jantung
terasa berdebar-debar sejak 2 minggu lalu. Pada pemeriksaan fisis, tanda-tanda vital dalam batas normal
kecuali suhu 38°C. hasil laboratorium menunjukkan leukosit 20.000 mm³.
Untuk menegakkan diagnosis penyakit tersebut diperlukan laboratorium …
a. LDH
b. SGOT
c. ASTO
d. CKMB Mass
e. Troponin I
54. Seorang laki-laki 50 tahun datang dengan nyeri kepala saat beraktivitas dan istirahat. Terdapat penonjolan
pembuluh darah di bagian temporal kepala dan terdapat pula banyak sel datia yang akan menjadi sel radang.
Diagnosis kasus tersebut adalah …
a. Takayasu disease
b. Giant cell arteritis
c. Raynaud disease
d. Buerger disease
e. Poliarteritis nodosa

57. Seorang wanita, 48 tahun datang ke poliklinik dengan keluhan nyeri dada sewaktu naik tangga dan
berkurang saat istirahat. Keadaan ini dirasakan sejak 3 bulan yang lalu. Ada riwayat hipertensi sejak 10 tahun
lalu dengan terapi captopril 2x25 mg/hari secara teratur. Lima bulan lalu sudah dilakukan pemeriksaan
ekokardiografi dan ditemukan adanya disfungsi diastolik. Hasil rekaman EKG seminggu lalu dalam batas
normal. Pertanyaan: Pemeriksaan yang ideal dan tepat untuk mengevaluasi keluhan nyeri dada pada penderita
ini adalah …
a. Uji latih jantung dengan beban
b. Ekokardiografi transesofageal
c. Elektrokardigrafi serial
8
d. Angiografi coroner
e. Monitoring Holter

58. Bakteri dengan hemolysis parsial dan sensitive Optochin adalah …

a. Streptococcus mutan
b. Streptococcus pyogenes
c. Streptococcus pneumoniae
d. Streptococcus agalactiae
e. Enterococcus faecalis
59. Seorang pria 35 tahun dengan kadar kolesterol 240 mg/dl, LDL 110 mg/dl, TD 160/90 mmHg datang ke
dokter untuk melakukan pemeriksaan kesehatan atas keinginan sendiri dan tidak dirujuk.
Upaya yang dilakukan orang ini adalah…
a. Pencegahan primer
b. Pencegahan sekunder
c. Pencegahan kuratif
d. Pencegahan tersier
e. Promosi kesehatan

63. Pasien laki-laki 55 tahun datang dengan keluhan nyeri dada yang bersifat intermittent dan menjalar ke
lengan kiri. Nyeri muncul ketika melakukan kegiatan dan menurun ketika beristirahat. Patomekanisme adanya
kelainan pada arteri coronaria yang berhubungan dengan keluhan nyeri pasien tersebut adalah :
a. Stable plaque
b. Unstable plaque
c. Fatty streak
d. Hyperplastic arteriosclerosis
e. Hyaline arteriosclerosis

64. Seorang perempuan 35 tahun berlibur ke negeri musim dingin mengeluh nyeri dada dan dibawa ke UGD.
Hasil EKG ada ischemia, dokter member nitrat dan “CCB”. 15 menit kemudian hasil EKG normal. Apakah
patomekanisme dari kejadian tersebut.
a. Adanya thrombus akut a. coronaria
b. Spasme a. coronaria
c. Rupture plak di a. coronaria

9
d. Atheroma di a. coronaria
e. Hipertensi berat bisa mengganggu suplai a. coronaria

67. Perempuan 69 tahun perokok berat datang ke dokter dengan keluhan bengkak pada tungkainya. Berjalan
10 meter harus istirahat, EKG tidak ada pembesaran jantung dan distention vena sentralis (DVS) normal. Pada
pemeriksaan, senyawa kimia yang ditemukan terkandung dalam sel-sel makrofag adalah …
a. Mukopolisakarida
b. Heparin
c. Hematin
d. Ferritin
e. Hemosiderin

70. Pria 36 tahun obesitas, perokok, nyeri dada 2 jam setelah bersepeda. Pemfis, EKG normal, obat yang
sesuai yaitu…
a. Beta-blocker
b. ARB
c. ACE-inhibitor
d. Aspirin dan nitrat
e. Digoxin

72. Seorang laki-laki 45 tahun ingin mendapat pendapat dari Anda, apakah ia mendapat risiko PJK atau tidak
dengan memperlihatkan hasil Lab : kolesterol total 140 mg/dl, HDL 20 mg%. apa pendapat Anda…
a. Bukan merupakan factor risiko karena kadar kolesterol total masih normal (<200 mg/dl)
b. Merupakan factor risiko karena rasio total kolesterol dan HDL = 7
c. Bukan merupakan factor risiko karena HDL masih normal
d. Perlu pemeriksaan LDL agar diketahui itu merupakan risiko PJK atau tidak
e. Perlu pemeriksaan EKG

74. Wanita 57 tahun diagnosis takikardia diberi terapi farmakologis pertama yang memperjang refrakter
nodus AV, meningkatkan tonus vagus dan menurunkan reaksi katekolamin. Obatnya adalah…
a. Digoxin
b. Aspirin
c. Lidokain
d. Diltiazem
e. Propranolol
75. Laki-laki 28 tahun datang ke klinik dengan keluhan lemah badan dan demam, kadang menggigil kadang
tidak. Mengeluh sesak, batuk, dan keringat malam. Tidak ada riwayat batuk menahun,tidak pernah tinggal di
10
daerah endemic malaria. Dia merupakan pegawai di perusahaan swasta. Ada riwayat infeksi gigi dan pernah
menindik telinga, tidak ada riwayat narkotik. Hasil pemfis TD 110/80 mmHg dan terdengar suara bising
jantung. Suhu 380C untuk diagnosis infective endocarditis menurut criteria duke adalah…
a. Satu kali diambil dan 1 kali negative
b. Diambil dua kali, satu kali positif
c. Dua kali dengan selang pengambilan 12 jam dengan salah satu positif
d. Dua kali selang 2 hari
e. Dua kali diambil bersamaan

77. Lapisan media yang memiliki otot polos yang tebal dapat ditemukan pada …
a. Arteri besar
b. Arteri sedang

c. Arteri kecil
d. Vena besar
e. Vena sedang

78. Pasien 63 tahun pernah mendapat serangan IMA 9 bulan yang lalu. Saat ini pasien sehat dan bugar,
meminta izin kepada dokter untuk mengikuti fun-aerobik di halaman PKM. Dokter memeriksa TTV dan
edukasi.
Kontraindikasi latihan pada pasien di atas adalah …
a. Penurunan TD sistolik (5 mmHg atau lebih) dari TD sehari-hari
b. TD sistolik >200 mmHg dan diastolic >100 mmHg
c. DM dengan kadar gula terkontrol
d. Batuk dan influenza
e. Penyakit alergi kulit

81. Seorang perempuan 14 tahun masuk UGD RS dengan keluhan jantung berdebar-debar sejak seminggu
yang lalu, keluhan ini disertai dengan sesak saat beraktivitas dan mudah lelas, pernah mengalai demam tinggi
dan batuk sebelumnya. Pada foto thorax simetris, JVP R+3, suhu 38°C.
Diagnosis kasus di atas adalah …
a. Infective Endocarditis (IE)

b. Pericarditis
c. Myocarditis
d. Temponade jantung
e. Penyakit jantung rematik

11
83. Seorang pria 59 tahun datang ke dokter dengan keluhan utama sesak napas, batuk pada malam hari, dan
edema pada tungkai. Ditemukan peningkatan tekanan vena 18 cm.
Untuk pemeriksaan penunjang maka dokter menggunakan serum pasien untuk melihat adanya peningkatan …
a. CKMB
b. AAT
c. BNP
d. LDH
e. Mioglobin
84. Seorang bayi 4 bulan diketahui susah menyusu dan terlihat sesak. Selain itu pada pemeriksaan fisis
ditemukan murmur pansistolik di anatar iga 3 dan 4 kiri tepi sternum. Anak tidak biru.

Diagnosis untuk kasus di atas adalah …


a. Tof
b. ASD
c. VSD
d. Stenosis aorta
e. Stenosis pulmonal

90. Kriteria minor untuk diagnostic EI


91. Pada auskultasi dada terdengar bunyi klik sistolik tinggi di dekat bungi S1 saat pasien berdiri. Dokter
mendiagnosis sebagai prolapse katup mitral.
Etiologi kasus prolapse di atas 90% berhubungan dengan …
a. Syndrome aport
b. Syndrome Ehler Dohler
c. Syndrome marfan
d. Syndrome crushing
e.

93. Perempuan 35 tahun masuk RS dengan keLUhan hemoptysis, sesak napas yang memberat sejak 1 bulan
yang lalu. Pemeriksaan fisis terdapat bising diastolic low-pitch dan bunyi opening snap yang jelas. Pada X-
Ray ditemukan kerleys B lines. Pada ekokardiografi terdapat dilatasi atrium kiri, orificium mitral 1,2 mm² dan
pada katup mitral juga ditemukan adanya kalsifikasi. Dokter mengatakan adanya regurgitasi mitral ringan.
Penangan lanjut untuk kasus di atas adalah …
a. Angiografi coroner
b. Operasi valvotomi
c. Profilaxsis antibiotic
d. Penggantian katup mitral

12
e. Valvuloplastin balloon katup mitral

100. Ciri dari vena besar pada tampakan mikroskopik adalah …


a. Terdapat tunika elastika externa
b. Memiliki lapisan yang jelas
c. Pada tunica adventisia terdapat vasa vasorum
d. Tunika intima terdapat serat elastika
e.

102. Seorang perempuan 60 tahun dengan obesitas, maka pencegahan primer yang dapat dilakukan adalah …
a. Menurunkan BB, ubah pola hidup, dan olahraga
b. Menurunkan BB dan diet 3 bulan
c. Menurunkan BB dan beri statin
d. Menurunkan BB dan berikan ACEI
e. Menurunkan BB dan beri statin+ACEI

2. Ace inhibitor mempengaruhi system bradikinin dengan


a. Increase degradation of bradikinin
b. Decrease degradation of bradikinin
c. Increase of the production of bradikinin
d. Increase the klierens of bradikinin
e. Direct effect on angiotensin II
3. Laki 19 tahun asymptomatic, terdapat murmur mitral regurgitasi pada echocardiograph, pada pemfis didapatkan
a. Early and midsystole murmur
b. Diminished first heart sound
c. Low pitched holosystolic murmur
d. Diminished forward stroke volume
e. Rapid decompensation with pulo edem
4. Pada siklus jantung, pernyataan dibawah ini terjadi pada fase pengisian, kecuali;
a. Tekanan atrium lebih tinggi dari tekanan ventrikel
b. Volume atrium tetap, tidak berubah
c. Katup mitral dan tricuspid terbuka
d. Katup aorta dan pulmonal tertutup
e. Gelombang p pada EKG Nampak
5. Seorang laki-laki 55 tahun, datang ke rumah sakit dengan keluhan sesak napas dan perasaan lelah. Pemeriksaan
fisis didapatkan TD 110/70mmhg, denyut jantung 110/menit, napas 30/menit. Pemeriksaan CXR menunjukkan
kardiomegali dan tanda-tanda edema interstitial. Pada pemeriksaan ekokardiographi menunjukkan ukuran ventrikel
kiri besar dari normal tet00api ketebalan dinding ventrikel kiri dalam batas normal, hanya gerakanya hipokinetik
menyeluruh. Apakah diagnose yang memungkinkan pada penderita?
a. Ischemic cardiomyopathy
b. Cor pulmonale chronicum
13
c. Congestive cardiomyopathy
d. Asymmetrica septal hyperthrophy
e. Idiopathic hyperthrophy subaorta stenosis

8. Dari pernyataan dibawah manakah menggambarkan action biological dari ACE?


a. Mempromosi degenerasi angiotensin II
b. Directly stimulate sintesis Aldosteron
c. Stimulate production of norepinephrine
d. Mengubah angiotensin I kepada angiotensin II
e. All of above

14. Komplikasi biopthotesis yang mana mungkin terjadi ?


a. Kebocoran paravalvular
b. Stenosis
c. Hemolisis
d. Embolisasi
e. Infark miokard
15. Perempuan 30 tahun riwayat IBS keluhan palpitasi tidak tentu sepanjang hari paling sering setelah minum
kopi. Sensasi utama degup loncat-loncat riang gembira yiha-yiha pada dada. Tidak pengsan. Tanda vital dan fisis
normal. Ekg irama sinus,kelainan lain tidak ada. Monitoring holter, kontraksi ventrikel premature 4x/min.
pengelolaan :
a. Terapi amiodarone
b. Terapi penyekat beta
c. Pemberian verapamil
d. Tidak diberi terapi
e. Rujuk elektrofisiologi
16. 15 years old boy rescending with his parents on a military have present with a fever of 38.6 degree celcius and
complaint of lower back pain knee and wrist pain. Tahe arthritis is not localized to any single joint. He gave a
history of severe sore throat several weeks earlier. Pemfis ada bonjolan kat elbow and wrist. Pemeriksaan lab
include negative blood cultur. Esr of 100 and antistreptolysin O elevated at this point. Nak bagi terapi apa kat
budak ni? Menyampah betul, menyusahkan hidup je.
a. Parenteral penicillin
b. Supportive care alone
c. Parenteral penicillin and aspirine
d. Parenteral penicillin and glukokortikoid
e. Parenteral penicillin, aspirin and diazepam
17. Jumlah presentasi pasien post MI tanpa factor risiko major yang boleh dimodifikasi adalah
a. <10%
b. 20-40%
c. 50%
d. 60-70%
e. 90%
18. Dalam keadaan normal, dari manakah awal dimulainya sistem konduksi jantung?
a. SA node
b. AV node
c. Bundle of his
14
d. Serat Purkinye
19. *Kasus mitral stenosis*
Wanita 38 tahun mudah lelah dan sesak nafas saat beraktivitas sejak 1 bulan yang lalu, TD 90/60mmhg,HR
95X/menit. Trdapat bunyi s2 keras dan opening snap.
Tindakan yg brsifat kardiologi intervensi pada penderita tersebut adalah?
a. Komisurotomi
b. Ballon mitral valve
c. Aortic Valve reconstruction
d. Mitral valve annuloplasty
e. Both aortic & mitral replacement
20. Substansi mana dibawah ini yang dihasilkan oleh sel endotel yang menyebabkan relaksasi?
a. ADP
b. Endothelin
c. Prostacyclin
d. Tromboxane A2
e. Superoxide anion

21. Antiarithmia mana yang membawa kepada atrial fibrilasi


a. Adenosine

b. Quinidine
c. Propafenone
d. Amiodarone
e. Atenolol

22. A 50yr old male with type 2 DM reports 3 months of exertional chest pain. His physical examination is notable
for obesity with a body mass index (BMI) of 32 kg/m2, BP 150/90mmHg and heard S4. No cardiac murmurs, and
no peripheral edema. Fasting glucose is 130 mg/dl and trigliserida 200 mg/dl.
Question : Which of the following is most likely in this patient?
a. Insulin resistance
b. Elevated HDL Cholesterol
c. Reduced serum endothelin level
d. Larger than normal LDL particles
e. Reduced serum homocysteine level
23. Indikasi penggantian katup aorta adalah
a. Dilatasi ventrikel ringan-sedang
b. Diameter sistol akhir < 50 mm
c. Gagal jantung diastolic
d. Endokarditis infeksi
e. Fraksi ejeksi <25 %
24. Hemoptysis pada penderita stenosis mitral disebabkan oleh
a. Adanya erosi saluran pernafasan atas
b. Pecahnya cabang arteri bronkialis
c. Akibat adanya anemia hemolitik
d. Terjadinya edema interstitial
e. Pecahnya alveoli

15
25. Lelaki 65 tahun, nyeri dada timbul tiba-tiba disertai sesak nafas mempunyai riwayat hipertensi dan DM. pemfis
menunjukkan kelainan pernafasan, kecemasan, dan keringat dingin. TD 180/ 110 mmhg. Nadi 170x/min dengan
irama regular. Pernafasan 40x/min. Terdapat ronki halus 2/3 basal kedua paru, desakan vena jugularis 12 cm.
terdengar S3 dan bising holosistolik derajat 2/6 di apex cordis. Foto torak menunjukkan pembesaran jantung dan
edema paru. Apakah diagnosis pasien ini.
a. Acute anterior myocardial infarction
b. Acute inferior myocardial infarction
c. Right ventricular infarction
d. Hypertensive heart failure
e. Cor pulmonal cronicum
26. Peningkatan nilai kolestrol dan LDL walaupun telah menjalani gaya hidup yang telah dimodifikasikan diberi
obat MHG KO-A reductase inhibitor dan nilai kimia darah dalam keadaan normal. Menjalani angiografi dan
didapat 70% stenosis pada ramus descendens anterior dan artery coronaria dextra. Apakah fungsi diberi mhg ko a
reductase inhibitor?
a. Mempengaruhi secara tidak langsung progresi ateroma
b. Menurunkan serum trigliserida dalam jangka panjang
c. Regrasi stenosis koronari yang ada
d. Prevent thrombus formation
27. 1 tahun infant, continous murmur

30. Gagal jantung diastolik paling sering disebabkan oleh?


A. Hipertensi sistemik B. IMA C. Kardiomiopati dilatatif D. Regurgitasi mitral berat E. Stenosis aorta yang sudah
berkalsifikasi

32. Pada gagal jantung sistolik terjadi perubahan hemodinamik akibat ?


a. Isi sekuncup bertambah dan volume diatolik akhir bertambah
b. Isi sekuncup berkurang dan volume diastolic akhir bertambah
c. Isi sekuncup berkurang dan volume diastolic akhir berkurang
d. Isi sekuncup bertkurang dan volume diastolic akhir normal
e. Isi sekuncup akhir normal dan volume diastolic akhir berkurang
33. Pemeriksaan lab dibawah ini yang paling sesuai dipakai untuk mendeteksi risiko kejadian penyakit
kardiovaskuler.
a. Pemeriksaan kadar kolestrol
b. Pemeriksaan kadar trigliserida
c. Pemeriksaan kadar HDL
d. Pemeriksaan kadar LDL
e. Pemeriksaan kadar apo A1

34. Seorang laki2 Umur 34 tahun . Nyeri Dada ketika naik tangga dan berhenti ketika beristirahat.
Gambaran histologi normal yang membedakan dengan vena besar
a. Tunika intima pada arteri, vena tdk
b. Pada vena terdapat Vasa vasorum , arteri tidak
c. tunika media Vena Lebih tipis dibanding arteri
d. Batas dinding vena tidak jelas sedangka arteri jelas
e. Tdk ada Lamina elastika pada arteri tapi ada pada vena

36. Repolarisasi sel myocardial ditentukan oleh aliran...


16
a. Outgoing sodium
b. Ingoing calcium
c. Outgoing potassium
d. Ingoing chloride
e. Ingoing sodium
37. Arteri intraventricularis posterior dipercabangkan
a. Arteri coronaria posterior
b. Arteri coronaria anterior
c. Arteri coronaria sinistra
d. Arteri coronaria dextra
e. Arteri coronaria media
38. Pembuluh darah yang bermuara ke atrium dextra adalah
a. Vena jugularis interna
b. Sinus coronaries
c. Vena pulmonalis
d. Vena subclavia
e. Vena renalis
39. Pasien mempunyai penyakit stenosis mitral murni disertai hypertrophy ventrikel kanan. Bunyi jantung ?
a. Bising diastolic kerana hipertro phy ventrikel kanan
b. Bising sistolik kerana hypertrophy ventrikel kiri
c. Bising sistolik kerana stenosis tricuspid
d. Bising sistolic kerana insufficiency triscupid

43. Mekanisma paling sering bagi arithmia cardiac


a. Trigger activity
b. Abnormal atomicity
c. Reentry
d. Early after depolarisasi
e. Parasistolic

47. Lelaki umur 55 tahun datang dengan keluhan sesak nafas dan berdebar. Tidak didapatkan riwayat infark
miokard tetapi ada riwayat hipertensi dan merokok. Pemfis TD 80/50 , HR 86x/min, pernafasan 26. Bising pada
apex jantung. Diagnosa?
a. Mitral regurgitasi dengan takikardi supraventrikuler
b. Mitral regurgitasi dengan takikardi ventrikuler
c. Stenosis aorta dengan fibrilasi ventrikuler
d. Kardiomyopathy dengan takikardi supraventrikuler

48. Seorang perempuan 20 tahun dengan keluhan berdebar-debar. HR 150x/min . Tiada riwayat sakit dan riwayat
pengobatan sebelumnya. Diagnosa?
a. Atrial fibrilasi
b. 3rd degree AV block
c. Ventrikel takikardi
d. Supraventrikel takikardi
e.

17
49. Laki 74 tahun penyakit CHF kerana miokard infark yang lama dengan keluhan sesak nafas sejak 2 jam yang
lalu. Pemfis dan chest x ray udem paru akut. Ekg narrow complex junction tachicardy. ST depresi pada V4-V6. TD
170/100. Pasien diber pengobatan nitrat, calcium channel blocker, digoxin, clorotiazede. Adakah HCT dan
flurosinamide jangka panjang diberikan pada pasien dapat member kesulitan hal-hal tersebut kecuali.
a. Hipomagnesemia
b. Hipocalcemia
c. Hiponatremia
d. Hipokalemia
e. Hiperuricemia

51. Struktur mediastenum inferior yang benar


a. Di posterior manubrium sterni
b. Di anterior vertebra thoracalis 6
c. Terdapat kelenjar timus
d. Dibagi menjadi 2 bagian
e. Terdapat arcus aorta
52. Pernyataan yang benar tentang proyeksi jantung dinding totaks yang benar adalah
a. Batas inferior kiri adalah kosta 5
b. Batas inferior kanan adalah ICS 5
c. Batas superior kiri adalah kosta 6
d. Batas superior kanan adalah kosta 3
e. Batas lateral kanan adalah linea medioclaviularis
53. Tipe kardiomyopathy yang dialami pasien tersebut diatas, penyebab utamanya selain akibat hipertensi lama
juga diduga banyak disebabkan akibat adanya mutasi pada gen-gen penting yang terdapat pada
a. Fibrosit
b. Endotel
c. Perimesium
d. Sarkomer
e. Myofilamen

54. Laki-laki 43 tahun datang kerumah sakit dengan keluhan sesak disertai nyeri dada, batuk dan demam juga
dikeluhkan. Pada pemeriksaan fisis ditemukan TD 180/100, pulsus paradoxus, nadi 112x/min, DVS R+3, ekg “low
QRS voltage”. Pasien ini diduga menderita efusi perikard dan hasil ct scan menyatakan adanya efusi perikard.
Dimanakah letak ruang yang mengalami efusi tersebut.
a. Tunika adventitia
b. Subendokardium
c. Endokardium
d. Myocardium
e. Epicardium
55. Jaringan apakah yag melapisi permukaan paling luar rongga yang mengalami efusi?
a. Mesotel
b. Pericardium
c. Endocardium
d. Tunika intima
e. Tunika adventitia

18
56. A case 10 years and 6 months of age visit OPD because pain of knee joint since 2 weeks ago. On physical
examination is heard apical murmur, x ray result with congestion pulmonal vascular marking. One of EKG result is
P mitral. The diagnose of this case according to WHO 2002-2003 is
a. Recurrent attack without rheumatic heart disease
b. Recurrent attack with rheumatic heart disease
c. Acute rheumatic fever
d. Rheumatic Chorea
e. Chronic valvulitis

57. A boy 10 years and 6 month of age hospitalized in pediatric department because heart failure due to acute
rheumatic fever. Lab finding WBC 17500/mm3, CRP 5 unit/ml, ESR 60 mm/h. The most appropriate management
of this case is
a. Bed rest of varying isn’t recommended
b. We have to postponed anti inflammatory treatment
c. Antibiotic administration to eradicated staphylococcus
d. We have to give BPG (Benzathine Penicillin G) during 10 months for secunder prophylaxis
e. Educate the patient and parents about the need to prevent subsequent staphylococci infection trough continuous
antibiotic prophylaxis

61. Seorang wanita 62 tahun masuk ke rumah sakit akibat mengalami pengsan berulang-ulang disertai kejang-
kejang, Tiada riwayat sakit dada, sesak nafas, cyanosis, mahupun edem tungkai. Pada pemeriksaan fisis, TD
180/80 mmhg dan nadi 40x/min teratur. Intensitas S1 pada apex jantung sangat bervariasi. Terdengar bising
sistolik ejeksi derajat 2/6 pada basis jantung. Diagnosis yang paling mungkin adalah
a. Brakikardi sinus dengan episode ventricular arrest
b. Blok AV kompolit dengan sindroma Adam Stokes
c. Fibrilasi atrium dengan blok AV parsial
d. Tumor otak dengan epilepsis sekunder
e. Blok AV 2:1 derajat 2

62. Pasien wanita keluhan nyeri ditungkai bawah, terasa demam tapi tidak terlalu tinggi, ade benjolan kecil pada
tungkai bawah sebelah dalam, sakit saat berjalan,lab menunjukkan ade tanda2 infeksi leukosit tinggi pada nilai
normal. Diagnosis?
a. Kaki gajah
b. Demam thyphoid
c. Tromboplebitis
d. Luka bakar derajat dua
e. sellulitis

66. Antiaritmia manakah yang tidak membuat prolog QT interval?


a. Quinidine
b. Lidocaine
c. Sotalol
d. Procainamide
e. Ibutilide
67. In which tissue involve in action potential ingoing calcium
a. Atrium
b. Ventrikel
19
c. AV Node
d. His Purkinje
68. penyebaran impuls pada system konduksi jantung paling lambat pada ?
a. serabut otot atrium
b. nodus AV
c. berkas HIS
d. Serabut purkinye
e. Serabut otot ventrikel
69. Which of the following antiaritmia agents is likely to cause the least torsades de pointes
a. Quinidine
b. Prokainamide
c. Flekainide
d. Ibutilide
e. Sotalol

70. Laki-laki 74 tahun dengan riwayat LVF akibat infak miokard datang ke UGD karena sesak nafas 2 jam
terakhir. Pemfis dengan x ray menunjukkan edem paru akut . Ekg tunjuk narrow complex conjuctional takikardi
pada rate 130x/min dengan 1 mm ST segment depression

pada V4-V6. TD 170/100 mmhg, Dj 32x/min. Mendapat pengobatan itrat, calcium channel blocker, digoxin dan
klorotiazide. Kemungkinan pembuluh darah koroner yang mengalami stenosis atau okulasi pada penderita ini
adalah
a. Left main artery
b. Left anterior descendens artery
c. Right coronary artery
d. Diffuse coronary artery
e. Posterior descending artery
71. Seorang laki-laki 26 tahun berobat ke poli jantung dengan keluhan utama cepat capek dalam 3 bulan terakhir.
Sebelumnya aktif berolahraga. Pemfis : TD 160/50 mmhg, nadi 90x/min kuat angkat, iktus kordis teraba di ICS VI
linea axillaris anterior sinistra dengan bising diastolic di daerah basis jantung. Diagnose sementara kasus ini adalah
a. Aorta stenosis
b. Mitral stenosis
c. Mitral regurgitasi
d. Aorta regurgitasi
e. Tricuspid stenosis
72. Seorang pria sementara bermain tennis, tiba-tiba dada sakit berat, disertai keringat dingin. Beberapa menit
kemudian penderita merasa lemah badan. Hasil EKG menunjukkan adanya ST depressi di lead II, III, dan aVF.
Pertanyaan: apa yang terjadi saat penderita bermain tennis?
a. Aterosklerosis
b. total AV blok
c. Ruptur plak
d. hipotensi tiba-tiba
e. hiperkolestrolemia
73. Seorang pasien didiagnosa sebagai gagal jantung kongesti berat. Pada pemfisterdengar ronchi di lapangan paru
kiri dan kanan, bunyi jantung s3 gallop, pada pemeriksaan CXR ditemukan udem paru bilateral. Untuk
mengurangi/ menghilangkan ronchi tersebut, maka diberikan
20
a. Beta blocker
b. Furosemid
c. Ace inhibitor
d. Aspilet
e. Oksigen
74. Lakilaki 52 tahun, gemuk masuk ugd denga sesak napas dan nyeri dada substernal, dirasakan 4 jam yang lalu.
Dari pemeriksaan fisis didapatkan tekanan darah 150/90 mmhg, tiada tanda-tanda gagal jantung. Ekg menunjukkan
adanya ST elevasi dan T inverted di sadapan precordial. Nyeri pasien beransur hilang setelah pemberian morfine
sulfate 2mg intravena. Terapi medis akut selanjutnya yang paling penting diberikan adalah
a. Dypiridamol
b. Propanolol
c. Cilostazol
d. Aspirin
e. Statin

75. Perempuan 62th dengan LBBB pada gambaran EKG masuk ICCU dengan keluhan nyeri substernal dan sesak
napas sejak 4 jam yang lalu. Laboratorium : serum tropinin-T meningkat. Dia menjalani tindakan kateterisasi
segera disertai angioplasty dan implantasi stain di arteri descendens anterior sinistra. Setelah 3 bulan dirawat, dia
kembali mengeluh nyeri dada. Pemeriksaan yg paling bermanfaat utk mencegah terjadinya kerusakan miokard
yang baru akibat infark miokard yang pertama adalah?
a. Echocardiogram
b. WBC
c. Serum troponin-T

d. Serum myoglobin
e. Electrocardiogram

81. Seorang laki-laki, 72 tahun dibawa masuk ke ruang UGD. Selama beberapa terakhir dia mengalami nyeri dada
yang berulang, batuk ringan dan sesak napas yang memberat bila berbaring. Hasil pemeriksaan ekokardiogram
menunjukkan fraksi ejeksi 35% dengan akinesia septum dan apeks ventrikel kiri.
Pertanyaan: Apa persangkaan diagnosis anda?
a. Edema paru akut
b. Aneurisma ventrikel
c. Kardiomiopati dilatatif
d. Reinfark miokard akut

e. Gagal jantung kiri akibat infark lama

85. Laki2 47 tahun dating ke rumah sakit utk evaluasi diabetis . tidak ada gejala pada pemeriksaan fisik . TD
140/90. KOLESTEROL 260 LDL 170 TRIGLISERIDE 200 GDP 145. Pendekatan terapi yang paling kurang
efektif untuk mencegah timbulnya PJK di masa akan datang?
a. Terapi statin
b. Terapi penyekat EKA
c. Terapi dengan o bat gemfibrozil
d. Control kadar gula darah secara ketat
e. Mengontrol tekanan darah di bawah 130/85

21
86. Pada mitral stenosis murni d sertai gagal ventrikel kanan maka kemungkinan akan terdengar bising jantung
yaitu:
a. Bising diastolic akibat hipertrofi ventrikel kanan
b. Bising sistolik akibat hipertrofi ventrikel kiri
c. Bising sistolik akibat insufisiensi tricuspid
d. Bising diastolic akibat stenosis tricuspid
e. Bising diastolic akibat hipertensi pulmonal
87. Seorang wanita 44 tahun sakit jantung reumatik dengan stenosis aorta menjalani ganti jantung dengan
bioprothesis . wanita tersebut tetap stabil selama 8 tahun setelah tindakan tersebut dan kemudian mengalami
penurunan toleransi terhadap aktivitas. APAKAH KEMUNGKINAN YG AKAN TERJADI PADA PASIEN INI?
A. Kebocoran paravalvular
B. Hemolysis
C. Stenosis
D. Embolisme
E. Infark miokard

89. SVT yang paling sering pada usia remaja adalah


a. Atrial ectopic takikardi
b. Atrial flutter
c. Jugularis ectipic takikardi
d. Avntr
e. Familial AF
100. Kelainan katup yang sering menimbulkan nyeri dada waktu istirahat malam adalah?
a. Stenosis aorta
b. Stenosis mitral
c. Insufisiensi aorta
d. Insifisiensi mitral
e. Stenosis pulmonal

101. Laki2 49 thn, ditermuakn diplodemia kadar kolesterol LDL tinggi meskipun telah melaksanakan modifikasi
gaya hidup. Dokter meresepkan obat HMG-Coareductase inhibitors…pemberiat obat yang diresepkan akan
memberi pengar7uh pada keadaan?
f. dapat menstabilkan lesi atreosklerotik yang sudah ada
g. berpengaruh tak langsung terhadap progresi atheroma

h. menurunkan serum trigliserida jangka panjang


i. meregresi stenosi coroner yang sudah ada
j. mencegah pembentukan thrombus

102. A 74yr old man with a long history of left ventricular failure secondary to several myocardial infarctions
comes to the emergency room acutely short of breath 2 hours after eating a large holiday meal. Pemfis and CXR
shows are consistent with acute pulmonary edema. ECG shows a narrow complex junctional tachycardia at a rate
of 130/min with 1 mm ST segment depression in lead V4-V6. BP is 170/100mmHg. His current medical treatment
includes nitrat, calcium channel antagonists, digoxin and chlorothiazide.
Otot jantung yang mengalami iskemia pada penderita diatas adalah...
f. Dinding anteroslateral
g. Dinding apicolateral

22
h. Dinding inferoposterior
i. Dinding high lateral
j. Ventrikel kanan

103. A 65yr old man developed an acute anterior myocardial nfarction several days ago. His course has been
complicated in the last 24h by a pericardial friction rub and pleuritic chest pain which has been difficult to control
with narcotics or steroids. He suddenly develops hypotension accompanied by marked DVJ and electromechanical
dissociation. No murmur are audibe.

Etiologi yang paling memungkinkan pada keadaan akut diatas adalah...


a. Regurgitasi mitral akut akibat ruptur m.papillaris
b. Rumpur septum ventrikel
c. Ruptur dinding jantung external/posterior
d. Perluasan infark miokard akut
e. Infark ventrikel kanan

104. Laki-laki 62 tahun, masuk RS dengan keluhan sesak terutama bila beraktifita disertai nyeri dada dan pasien
sempat pingsan sebelum dibawa ke RS. Pasien memiliki riwayat hipertensi lama dan berobat tidak teratur. Pada
pemfis TD 190/100, batas jantung melebar, ronkhi basah halus basal paru bilateral. EKG ada gambaran LVH,
echocardiagram memperlihatkan adanya disfungsi diatolik. Oleh dokter pasien didiagnosis mengalami
kardiomiopati tipe hipertrofik.
Jaringan jantung yang mengalami hipertrofi pada kasus diatas apabila berada dalam kondisi normal, manakah
pernyataan dibawah ini yang paling benar?
f. Tidak ada fascia adherent
g. Terdapat diskus interkalaris
h. Memiliki inti yang terletak di perifer
i. Memiliki gap junction pada segmen horizontal
j. Aparatus golgi lebih banyak pada otot ventrikel

105. A pediatrician examines a 2 months old infant who had been born at term. The pedatrician heard a contionous
murmur at the upper left sternal border. The peripheral pulses in all extremities are full and show widened pulse
pressure. Which of the following in the most likely diagnose?
f. Coarctation of the aorta
g. Patent ductus arteriosus
h. Ventricular septal defect
i. Persistent truncus arteriosus
107. The major cell type of the normal coronary artery intima is the:
o. Macrophage
p. Smooth muscle cell
q. Lymphocyte
r. Endothelial cell
s. Foam cell

21. Arteri descendens posterior dipercabangkan oleh ...


a. a. coronaria posterior
b. a. coronaria anterior
c. a. coronaria dextra
d. a. coronaria sinistra

23
e. a. coronaria media

25. Pada siklus jantung, peristiwa ini terjadi pada fase diastolik, kecuali:
a. Tekanan atrium lebih tinggi dari tekanan ventrikel
b. Volume atrium tetap, tidak berubah
c. Pembukaan katup mitral dan trikuspid
d. Penutupan katup aorta dan pulmonal
e. Gelombang P pada EKG

54. Repolarisasi sel myocardial ditentukan oleh aliran...


a. Outgoing sodium
b. Ingoing calcium
c. Outgoing potassium
d. Ingoing chloride
e. Ingoing sodium

11. Akulstasi dada ada bunyi klick sistolik tinggi dekat s1 bunyi saat pasien berdiri. Dokter diagnose ada prolapsus katup
mitral. Ternyata komponen matriks ekstraselular normal diganti jaringan ikat abnormal ** tk ingat
A. Laminin & fibronektin B. Entaktin & fibrin C. Elastin & fibrin D. Fibronektin & entaktin E. Elastin & tk igt
12. Etilogi kasus di atas pola perhubungan somatosa d mana sindrome d bawah 90% patologi prolapsus mitral
A. Sindrome aport B. Sindrome elias ambos**tksure C. Sindrome marfan D. Sindrome crushing E. Sindrome **tkigt maaf..
pjg..

14. Selain preparat thrombolitik yang digunakan atas preparat thrombolitik lain yang membedakan sasaran kerjanya. Dalam
mengaktifkan plasminogen yang membedakan adanya atau tidak ikatan plasminogen dengan senyawa A. Glikoprotein B.
Fibrin C. Tromboxan D. Trombosit E. Serotonin

16. Treadmill test positive when A. ST depresi >2 mm pada AVR B. ST depresi 1 mm pada V1 C. ST depresi 1 mm pada V2 D.
ST depresi > 2 mm pada AVr E. ST depresi >2 mm pada V2-317

41.sindrom marfan predipasi terjadi kasus diatas kerana adanya degredasi jaringan elastika otot tunika media pembuluh
darah tersebut ini kerana maturasi gene tersebut
a.fibronectin
b.fibrilin
c.kolagen
d.lamina
e.entraktil

44. Seromaker yang spesifik untuk mendeteksi adanya kerusakn miokard, bila onset nyeri dada sekitar 1 jam lalu adalah : A.
CK-MM B. CK-MB C. CK-BB D. SGOT E. SGPT

49.Which of the following antiarrhythmic agents may promote AF?


A.Adenosine
B.Quinide
C.Propafenone
D.Amiodarone.
E.Atenolol.
50. Which of the following antiarrhythmic agents is list likely to cause torsades de pointes?
A. Quinide
B. Procainamide.
C. Flecainide.
D. Ibutilide.

24
E. Sotalol
51 kerusakan membrane plasma otot jantung pada penderita infark miocard akut diakibatkan oleh peran enzim?
A fosfruktokinase
Bfosfodieterase
C fosfokinase
D fosfolipase
61) Laki- laki, 52tahun, gemuk, masuk UGD dengan sesak napas dan nyeri dada substernal , dirasakan 4 jam yang lalu. Dari
pemeriksaan fisis didapatkan tekanan darah 150/90 mmHg, tiada tanda-tanda gagal jantung. EKG menunjukkan adanya ST
elevasi dan T inverted di sadapan pericordial. Nyeri pasien beransur hilang setelah pemberian morfine sulfat mg intravena.
Pertanyaan: Terapi medis akut selanjutnya yang paling penting diberikan adalah:
a) Dypiridamol
b) Propanolol
c) Cilostazol
d) Aspirin
e) Statin

62) Laki- laki, 36 tahun, sesak napas. Tiada riwayat hipertensi, diabetes mellitus dan merokok . Pada pemeriksaan fisis, sakit
sedang , tekanan darah 130/90, Nadi : 100 kali per minit , ampltudo nadi melebar . Kedengaran bbuyi bising diastolik derajat
III/IV di basis. CXR menunjukkan kardiomegali.
a) Regurgitasi Aorta
b) Regurgitasi Pulmonal
c) Regurgitasi Tricuspidalis
d) Stenosis & Regurgitasi Aorta
e) Stenosis & Regurgitasi Pulmonal

66) Bayi pemeriksaan postnatal. 1 hari lahir ada bising jantung sistolik yang halus di ICS kiri atas. Pemeriksaan fisis lain tidak
ada kelainan. Echocardiography rencana lagi 2 minggu. Hari ke-5, bayi pucat dan lesu. Pemeriksaan fisis, nampak perfusi
perifer menurun dengan perubahan warna kulit & pengisian kembali kapiler lambat. Denyut femoral menghilang. Denyut
pada extremitas atas tetap teraba dan jelas. Diagnose?
A. Coarctation of the aorta
B. Patent ductus arteriosus
C. Congenital aortic stenosis
D. Congenital pulmonary stenosis
E. Idiopathic hypertrophic subaortic stenosis

68. Perempuan 30 tahun, riwayat IBS, keluhan palpitasi, tak tentu sepanjang hari, paling sering setelah minum kopi. Sensasi
utama degup loncat loncat pada dada. Tidak pingsan. Tanda vital n fisis normal. EKG irama sinus, kelainan lain tidak ada.
Monitoring holter, kontraksi ventrikel premature,4x/menit.
Pengelolaan
A. Terapi amiodarone
B. Terapi penyekat beta
C. Pemberian verapamil
D. Tidak diberi terapi
E. Rujuk elektrofisiologi
69) Keluhan mual, muntah,

71. Seorang laki-laki 47 tahun datang ke klinik untuk evaluasi diabetis. Tidak ada gejala didapatkan pada pemeriksaan fisis.
Pemeriksaan tekanan darah 140/90 mmHg, kolesterol total 260 mg/dl, LDL 170 mg/dl, serum trigliserida 200 mg/dl dan gula
darah puasa 145 mg/dl. Manakah antara berikut merupakan terapi yang paling kurang efektif dalam mencegah penyakit
jantung koroner dan strok pada pasien? a. Terapi statin b. Terapi obat gemfibrozil c. Kontrol pemberian gula darah dengan
ketat d. Mengawal tekanan darah kurang 130/85 mmHg e. Terapi ACE inhibitor

25
72. Seorang perempuan 62 tahun, dengan left bundle branch block pada EKG, masuk ICCU dengan nyeri substernal dan sesak
nafas selama 4 jam yang lalu. Troponin T meningkat dan dibuat tindakan kateter segera dengan angioplasti dan implantasi
stent pada arteri descendent anterior kiri. Selepas 3 bulan, pasien datang kembali dengan nyeri dada. Pemeriksaan manakah
yang bermanfaat untuk mendeteksi kerusakan miokard akut yang baru setelah infark miokard pertama? a. Ekokardiogram b.
Serum troponin I c. Serum troponin T d. Serum myoglobin e. EKG

74. 28 years old man is seen in the clinic for rutin physical examination. he has no complains and is asymptomatic with no
episode of syncope, and he has no family history of cardiac disease.II/IV sistolik murmur is heard best and the right lower
sternal border. the murmur is accentuated with inspiration and decreases valsalva. the murmur is was likety due
a. flow murmur
b. aortic regurgitation
c. mitral regurgitation
d, tricuspid regurgitation
e hypertrophic cardiomeopathy
75) Seorang perempian berumur 30tahun dengan stenosis mitral telah memjalani operasi kumisurotomi. Pasca operasi yang
berhasil dia masih harus memperoleh antibiotik untuk mencegah timbulnya endocarditis infektif atau demam reuma
kambuhan.walaupun demikian,trrkadang ditemukan gejala sisa operasi komisurotomi mitral tersebut. Gejala sisa apa yang
paling umum dijumpai pada penderita tersebut?
A) mitral regurgitasi B) AV blok C) LV failure D) pulmonary atherosclerosis E) retstenosis katup mitral

77. A patient is admitted to the emergency department following a drug overdose. He is needed to have severe tachyenlin.
He has been receiving theraphy for hypertension and agina drug often cause tachyardia is
A. Kaptoril
B. Nifedipin
C. Dilitiazem
D. Verapanik
E. Propranolol
78. Patient with long standing diabeti renal disease, hypertension and hyperkulamia has recent onset of heart failure. Which
following agent would be least harmfullin in this patient.
A. Cuptopril
B. Valsartan
C. Triarmterene
D. Spiroholactone
E. Hydrochorthiazide

81. A 10 years 6 months boy hospitalized because of heart failure due to acute rheumatic fever. Lab; WBC:17500 mm3,
CRP:5 ul/ml ESR:60ml/hr. The best management for this boy : A. Bed rest varying duration is not recommended B. Postponed
anti-inflammatory treatment C. Antibiotics to eradicate staphylococcus D. BPG during 10 months for secondary prophylaxis
E. Educate patient and parents to prevent subsequent infection of streptococcus with antibiotics prophylaxis

82. Which factors that influences the ductus closure? A. Indomethacin concentration is low in circulation B. prostaglandin
level is low in circulation C. Jaundice baby D. Aterm baby E. Apgar level 8/10 after birth

83. A baby 7 day of age was born spontaneously, crying soon after birth with apgar score 7/10, body weight 3900 grams.
Eight days later the baby looks cyanotic and we order xray examination. We expected the shape of the heart on x ray more
likely :
A. egg shaped
B. boot shaped
C. peer shaped
D. carpet shaped
E. snowman like appearance

26
84. A baby days of age was born spontaneously without crying soon with apgar score 5/10. Body weight 1900 grams. We
order chest x-ray, ECG and echocardiography examination. The result of echocardiography point out moderate PDA. Which
of this statement below correlated with PDA :
A. On chest x-ray seen apex upturned
B. There is a narrow pulse pressure
C. There are LA and LV dilatation
D. Mostly seen in premature baby
E. On ECG we got p pumonal

88. 9 years old has been hospitalize due to dyspnea. Pemsis: apical murmur. Xray: congestion pulmonal vascular marking.
One of ECG show P mitral. On echocardiogram, p mitral and aortic regurgitation. Berapa lama treatment yg diberi bagi
mengelakkan serangan berulang?
A.1 year B.3years C.5 years D. 10 years E. Long life

89. Anak lelaki umur 2 tahun kontrol di poli jantung anak kerana sianosis yg dialami sejak lahir. Pada pemeriksaan fisis
didapatkn bunyi jantung 2 tunggal dengan bising ejeksi sistolik pada ics 2-3 pada paresternal kiri. Pada foto toraks lateral
didapatkan bayangan retrosternal space menghilang. Diagnosisnya adalah? A. VSD B. ToF C. PDA D. ASD E. Tricuspid atresia

90. A 7years old child returns for follow up. He is pink and healthy looking with normal vital signs, pulses and precordial
activity. However the anteroposterior diameter of her chest is somewhat increased. The lungs are clear. S1 is normal but S2
is fixed and widely split. A 2/6 medium frequency systolic ejection murmur is heard at the upper left sternal border and all
along the border, it is also heard well in the lung fields. There is also an early 2/6 diastolic murmur, almost decrescendo at
the lower at the lower left sternal border. The liver is 1cm below the right costal margin. What is the murmur at the upper
left sternal border most likely caused by? A. Mitral insufficiency B. Pulmonary stenosis C. Tricuspid stenosis D. Mitral stenosis
E. Aortic stenosis

21. Tanda klinis yang bukan merupakan gejala atau komplikasi ToF adalah :
A. Sianosis
B. Abses otak
C. Gagal tumbuh
D. Cyanotic spells
E. Gagal jantung kongesti.

22. A 7 month-old boy comes to the hospital because of cough and hard to breath. These symptoms occur frequently since he
was a 2-month baby. On physical examination murmur is heard grade 4/6 p.m. at LSB4 spread to RSB, axillary and
suprasternal.
Question: Which of the following is the most likely diagnosis?
A. ASD : ejeksi sistolik
B. PDA :murmur continuous
C. TOF :murmur ejesksi sistolik
D. TGA
E. VSD. : sistolik , ada batuk juga

23. Seorang anak berumur 6 tahun secara tak sengaja ditemukan adanya bising jantung sewaktu pemeriksaan rutin untuk
menilai status kesehatannya oleh dokter keluarga. Sang anak sebaliknya tanpa keluhan dan dapat melakukan kegiatan fisik
sehari-hari. Pada pemeriksaan fisis lainnya semua dalam keadaan normal, kecuali adanya bising sistolik yang halus yang
terdengar pada ruang interkostal-4 linea para sternal kir, yang berubah-ubah intensitasnya pada perubahan posisi tubuh.

Pertanyaan: Kelainan mana yang paling mungkin yang menyebabkan bising jantung semacam ini?
A. Venous hum
B. Still’s murmur.
27
C. Supra-clavicular bruit
D. Pulmonary flow murmur
E. Peripheral pulmonary stenosis of newborn

24. Seorang bayi umur 2 bulan dibawa ke rumah sakit kerna sesak napas dan susah makan dalam 2 minggu terakhir. Pada
pemeriksaan fisis bayi kemerahan dengan takhipnea dan daearh subkostal tertarik kedalam. Nadi perifer normal. Pemeriksaan
pericardial menunjukkan adanya impuls ventrikel kiri, bunyi jantung normal dan terdengar suatu bising sistolik kasar di sela
iga kiri-4. Hati membesar 2 cm dibawah kosta kanan.
Pertanyaan: Diagnosis yang paling mungkin adalah?
A. Atrial septal defect with mitral valve prolapse

B. Ventricular septal defect with heart failure. :bising sistol :VSD, hati membesar :gagal jantung kanan
C. Atrial septal defect with heart failure
D. Sub-aortic ventricular septal defect
E. Eisemenger’s syndrome

25. A small PDA usually causes no disability during childhood. Which of the following is a dangerous complication of the
small PDA that can rapidly convert this benign lesion into a serious one?
A. Pneumonia
B. Hypertension
C. Infective endarteritis. :endocarditis maksudnya
D. Pulmonary embolism
E. Aortic valve prolapsed *sy bingung

26. Severe pulmonic stenosis may be associated with central cyanosis. Which of the following is the usual mechanism of this
finding?
A. Polycythemia
B. Right ventricular failure
C. Anomalous pulmonary venous drainage
D. Concomitant pulmonary arteriovenous fistula
E. Unsealed foramen ovale with right atrial hypertension.
27. Seorang laki-laki 67 tahun dirawat rumah sakit dengan Infark miokard akut anterior dan diberi terapi trombolitik. Tiga hari
kemudian dia mengeluh nyeri dada yang makin bertambah bila baring telentang. Pada pemeriksaan fisis, semua normal
kecuali terdengar gesekan perikard. EKG menunjukkan perubahan yang sesuai untuk IMA anterior namun terlihat adanya
depresi segmen_ST yang baru dan elevasi segmen-ST diseluruh sandapan extremitas.
Pertanyaan: Diagnosis yang paling memungkinkan adalah :
A. Reinfarction
B. Viral infection
C. Pulmonary embolus
D. Post-MI pericarditis. :nadi meningkat saat berbaring
E. Dissecting aneurysm

28. Seorang laki-laki 61 tahun dimasukkan ke ICCU dengan diagnosis non-STEMI. Pada hari berikutnya dia mengalami
bradikardia tapi tanpa keluhan. Tekanan darahnya 125/85 mmHg, nadi 50/menit, dan pada pemeriksaan fisis, bunyi jantung
normal tanpa adanya bunyi tambahan atau gesekan perikard. Gambaran EKG nya berubah.
Pertanyaan: Perubahan EKG yang mana yang merupakan indikasi pemasangan pacu jantung pada penderita ini?
A. Persistent bradycardia
B. First degree AV block
C. New right bundle branch block
D. Second-degree AV block Mobitz type I
E. Left BBB and second degree AV block Mobitz type II.

28
29. Pemeriksaan auskultasi jantung pada seorang pemuda 17 tahun menunjukkan adanya peningkatan intensitas S2 komponen
pulmonal . Dia mengeluh sesak napas waktu bergiat tetapi tidak ada gejala-gejala yang menonjol pada jantung dan paru.
Pertanyaan: Kelainan mana dibawah ini yang paling mungkin sebagai penyebab sesak napas pada penderita ini?
A. Aortic stenosis
B. Pulmonary stenosis : sesak nafas bukan karna bendungan paru, tapi karna sedikitnya darah yg ke pulmo
C. Myocardial infarction
D. Systemic hypertension
E. Pulmonary hypertension.

31. A 73-year-old man has angina pectoris on exertion, but an angiogram reveals noncritical stenosis of the coronary arteries.
This occurs most frequently with abnormality of the valve.
Question: Which of the following valvular heart disease is the most likely cause ?
A. Mitral stenosis
B. Aortic stenosis.
C. Mitral insufficiency
D. Pulmonary stenosis
E. Aortic insufficiency

32. Seorang gadis 17 tahun datang kerumah sakit dengan keluhan sesak napas bila bergiat, tak ada batuk maupun gejala
mengik. Pada pemeriksaan fisis, terdengar bunyi S2 pecah menetap disertai bising sistolik ejeksi derajat 3/6 yang terdengar
jelas di sela iga kiri. Pemeriksaan ekokardiogram dilakukan untuk memastikan diagnosis.

Pertanyaa: Mekanisme patofisiologi yang mana dibawah ini yang paling cocok untuk menjelaskan keadaan klinis diatas? :
VSD
A. The left ventricle is enlarged
B. The systemic blood pressure is elevated
C. Pulmonary blood flow is equal to systemic blood flow
D. Pulmonary blood flow is less than systemic blood flow
E. Pulmonary blood flow is greater than systemic blood flow.

34. Seorang perempuan 63 tahun yang mengkonsumsi digitalis untuk terapi fibrilasi atrium kronis yang dideritanya mengeluh
capek, mual dan anoreksia. Nadi regular 50/menit, dan bunyi jantung, dada, abdomen normal. Pada pemeriksaan EKG tidak
ada gelombang P tapi kompleks QRS sempit dan reguler.
Pertanyaan: Tindakan lanjut apa yang paling tepat pada pengelolaan penyakit penderita?
A. Addition of a beta-blocker
B. An increase in digitalis dose
C. Complete cessation of digitalis. :cepat akhiri ! sudah ada gejala yg diakibatkan efek samping digitalis
D. Withdrawal of digitalis for one dose
E. Addition of a calcium channel blocker

35. A 47-year-old man is found to have edema, ascites, and hepatomegaly. The examination of his neck vein reveals of 5 cm
from sternal angel at 30˚ position. Heart size on x-ray is normal.

Question: Which of the following etiologies is not a possible explanation for this syndrome ?
A. Tuberculosis
B. Unknown cause
C. Rheumatic fever. :menyebabkan penyakit katup
D. Previous acute pericarditis
E. Neoplastic involvement of the pericardium

37. Three months after an anterior MI, a 73-year-old man has a follow-up ECG. He is clinically feeling well with no further
angina symptoms. His ECG shows Q waves in anterior leads with persistent ST-segment elevation.
Question: The current ECG is most compatible with which of the following diagnosis ?
29
A. Acute infarction
B. Early repolarization
C. Ventricular aneurysm. : ST elevasi : infark, aneurisma
D. Hibernating myocardium
E. Silent myocardial infarction

38. A 79-year-old man presents with syncope. On physical examination, he has a slow upstroke in his carotid pulse and a
diamond-shaped systolic murmur at the base. His chest is clear.
Question: Which of the following findings is his CXR most likely to reveal ?
A. Displaced apex
B. Left atrial hypertrophy
C. Right ventricular dilatation
D. Normal overall cardiac size.
E. Stenosis of the proximal ascending aorta :ingat, diamond shaped systolic itu stenosis aorta

39. Seorang laki-laki 62 tahun datang ke rumah sakit dengan keluhan sesak napas yang makin bertambah, dan akhir-akhir ini
susah baring telentang. Pada pemeriksaan fisis ditemukan JVP 8 cm dan terdengar S3, ada edema pretibial dan krepitasi kedua
basal paru.
Pertanyaan: Kelainan mana dibawah ini yang paling berperan menimbulkan retensi cairan sehingga timbul gejala seperti
disebutkan diatas?
A. Decreased renin
B. Increased estrogen
C. Increased aldosterone. :peningkatan RAA yg bs menyebabkan gejala pd penyakit cardiovascular
D. Decreased vasopression
E. Increased growth hormone

40. A 60-year-old woman presents with symptoms of weight loss, anxiety, and palpitations. On examination, she has a thyroid
goiter.

Question: Which of the following is the most likely cardiac finding ?


A. Aortic insufficiency
B. Pericardial effusion
C. Decreased cardiac output
D. Prolonged circulation time
E. Paroxysmal atrial fibrillation. :hiperthyroidisme : palpitasi

41. Seorang perempuan 70 tahun ditemukan mempunyai nadi yang irregular pada pemeriksaan rutin. Dia tidak mengalami
keluhan yang baru baik waktu istrahat maupun waktu bergiat. Pada pemeriksaan EKG, tidak ada gelombang P, interval RR
tidak teratur dengan laju jantung 70/menit. EKG sebelumnya, 4 tahun yang lalu iramanya adalah irama sinus.

Pertanyaan: Tindakan/obat apa yang paling tepat diberikan sebagai tindak lanjut pada penderita ini?
A. Aspirin
B. Beta-blocker
C. Cardioversion
D. Anticoagulant. :heparin, walfarin : AF kronis (kan sdh 4 tahun, bedanya dlu msh ad gelombang P (sinus), skrg sdh asinus),
orang AV lbih bsar peluang buat kena strok, shg dikasih antikoagulan
E. Antiarrhythmic therapy

42. Seorang perempuan 22 tahun mengeluh berdebar-debatr, nyeri dada dan sedikt lelah sewaktu bergiat. Pada pemeriksaan
fisis tekanan darah 130/85 mm Hg. Ekokardiogram menunjukkan adanya prolaps katup mitral.
Pertanyaa: Temuan fisik yang paling umum dijumpai pada penderita diatas adalah?
30
A. Diastolic click
B. Diastolic rumble
C. Aortic regurgitation
D. Late systolic murmur.

43. A 65 year-old man complains of postural hypotension with dizziness. His blood pressure is 110/80 mm Hg supine and
85/70 mm Hg standing (after 2 minutes), the pulse rate remains the same at 80/min.
Question: Which of the following is the most likely diagnosis ?
A. Thyrotoxicosis
B. Diuretic therapy
C. Venous varicosities
D. Essential hypotension
E. Possibility of diabetes mellitus.

46. A 75-year-old woman with hypertension develops fatigue and dyspnea on exertion. Her blood pressure is 160/60 mm Hg
and pulse 80/min. The second heart sound is diminished and there is an early diastolic murmur that radiates from the right
sternal border to the apex.
Question: Which of the following characteristic arterial pulse findings in this patient ?
A. Pulsus tardus :Aorta stenosis
B. Dicrotis pulse : LVH, tiphoid, dehidrasi, paling mungkin pd yg demam
C. Pulsus alternans : AR
D. Pulsus paradoxus :temponade jantung, perikarditis kostriktivam emfisema paru yg berat dan asma bronkialis
E. Hyperkinetic pulse.

47. Seorang laki-laki 64 tahun dengan riwayat pernah mengalami infark miokard dua kali datang kerumah sakit dengan sesak
napas dalam keadaan istrahat dan susah berbaring telentang. Tekanan darah 95/70 mm Hg, nadi 100/menit, dan JVP 8 cm pada
posisi supine. Apex kordis melebar kebawah lateral kiri, bunyi jantung n ormal, tetapi terdengar S3. Diagnosis anda adalah
kardiomiopati iskemik (HOCM).
Pertanyaan: Apa jenis pulsus yang ditemukan pada penderita ini?
A. Dicrotic pulse.

B. Bisferiens pulse
C. Pulsus bigeminus
D. Hyperkinetic pulse
E. Delayed femoral pulse

48. Seorang perempuan 45 tahun mengalami sesak napas sewaktu bergiat dan terasa cepat lelah. Pada pemeriksaan fisis
terdewngar bising sistolik ejeksi yang jelas terdengar pada sela iga kiri, dan intensitas bising meningkat pada posisi berdiri.
Pada pemeriksaan palpasi teraba impuls ganda di apeks kordis.
Pertanyaan: Berdasarkan temuan fisis, apa diagnosis yang paling mungkin?
A. Aortic stenosis
B. Pulmonary stenosis
C. Mitral valve prolapse
D. Chronic mitral regurgitation
E. Hypertrophic obstructive cardiomyopathy. (?)

49. Seorang perempuan 42 tahun salah seorah eksekutif pada suatu perusahaan besar, datang ke klinik untuk evaluasi tahunan.
Dia sangat kuatir dengan kemungkinan adanya factor risiko untuk terserang penyakit jantung dimasa datang karena salah
seorang mitranya telah pernah mengalami serangan angina. Tidak ada riwayat adanya penyakit sebelumnya, tidak pernah
merokok. Pada pemeriksaan laboratorium, T-chol 240 mg/dL, HDL 55 mg/dL, LDL 160 mg/dL, and TG 140 mg/dL.
Pertanyaan: Pengelolaan yang paling tepat diberikan untuk dislipidemia pada penderita ini adalah?
A. Nicotinic acid

31
B. Lifestyle modification.
C. Fibric acid derivatives
D. Bile acid-binding resins
E. HMG CoA reductase inhibitors : statin

50. Seorang laki-laki 65 tahun dengan riwayat sebelumnya menderita infark anterior datang ke rumah sakit untuk pemeriksaan
rutin. Pada pemeriksaan fisisterdengar bising sistolik yang paling jelas terdengar di apeks kordis dan menjalar ke axilla.
Kompressi eksternal sementara pada kedua lengan dengan manset 20 mmHg diatas tekanan sistolik ternyata meningkatkan
intensitas bising.
Pertanyaan: Berdasarkan sifat bising yang berubah pada waktu kompressi, mak diagnosis yang paling mungkin adalah?
A. Pulmonary stenosis
B. Mitral regurgitation.
C. Barlow’s syndrome
D. Tricuspid regurgitation
E. Hypertrophic cardiomyopathy

51. Seorang perempuan 28 tahun batu saja mengeluh nyeri dada yang berubah intensitasnya pada perubahan posisi tubuh.
Nyeri bertambah bila berbaring telentang dan berkurang bila duduk. Saat ini nyeri mulai berkurang namun timbul rasa sesak
napas dan edema tungkai. Pada pemeriksaan fisis tekanan darah 85/60 mm Hg tetapi pada inspirasi tekanan darah sistolik
menurun lebih darai 10 mmHg, nadi lemah 110/menit, dan bunyi jantung terdengar jauh. JVP 7 cm dengan Kusmaull’s sign
negatif.

Pertanyaan: Diagnosis yang paling mungkin adalah ?


A. Cardiac tamponade. :kusmaul sign positiv , suara jantung redup, penurunan tekanan sistolik saat inspirasi, peningkatan
JVP, takikardi, nyeri betambah bila berbaring, berkurang jika duduk

B. Constrictive pericarditis :nyeri saat berubah posisi, berkurang bila bersandar ke depan
C. Restrictive cardiomyopathy
D. Right ventricular myocardial infarction
E. Inferior myocardial infarction with hypotension

52. Seorang perempuan 60 tahun yang kuatir dengan factor risiko koroner yang kemungkinan disandangnya karena dia sudah
dalam keadaan menopause. Pada pemeriksaan tidak dijumpai adanya gejala penyakit kardiovaskular dan satu-satunya factor
risiko yang disandangnya adalah hipertensi sejak 5 tahun lalu yang terkontrol baik. Laboratorium, fasting T-chol is 240
mg/dL, HDL 55 mg/dL, LDL 160 mg/dL, and TG 140 mg/dL.
Pertanyaan: Tindakan/obat yang mana yang paling tepat disarankan kepada penderita ini?
A. Simvastatin
B. Gemfibrozil
C. Nicotinic acid
D. Lifestyle modification.
E. Hormonal replacement with estrogen

53. A 54-year-old man with diabetes has a persistently elevated blood pressure averaging 150/90 mm Hg. He has complication
of peripheral neuropathy and a urinalysis is positive for microalbuminuria.
Question: For his elevated blood pressure which of the following is the most appropriate medication ?
A. Thiazides
B. Clonidine
C. Beta-blockers
D. ACE-inhibitors. : hipertensi yang disertai DM
E. Calcium channel-blockers

54. Wanita 55 tahun mengeluh sering pusing. Tekanan darah 150/95 mmHg, hepar teraba 2 jari bawah arkus kosta kanan.
Hasil pemeriksaan EKG menunjukkan ritme sinus dengan denyut nadi 90/menit, LVH. Gambaran foto thorax: elongatio aorta
32
dan kardiomegali, corakan paru kasar . Pemeriksaan laboratorium: ureum 80 mg/dL, kreatinin 3,5 mg/dL. Pada CT scan
kepala: ditemukan edema serebri.
Pertanyaan: Organ-organ yang termasuk “ target organ damage” pada kelainan diatas adalah?
A. Otak dan jantung
B. Otak, jantung dan aorta
C. Otak, jantung, aorta dan ginjal.
D. Otak, jantung, aorta, ginjal dan hati
E. Otak, jantung, aorta, ginjal, hati dan paru

55. A 75 year old woman returns to the clinic for follow-up. Blood pressures were 160-170/70-80 at presentation and are now
140-150/60-70 on amlodipine 5 mg/day. She asks if she can stop her medication because she thinks it’s making her tired and
her feet are swelling, making it difficult to wear her best shoes. BP 149/62. P 84.
Question: What should you tell her?
A. Stop her medication
B. Reduce the dose of amlodipine
C. Combine amlodipine with HCTZ :hidro clorotiazid (diuretik) : utk buang cairan dan kompensasi efek samping ccb juga
D. Recommend salt restriction and weight loss.
E. Change her medication with other calcium antagonist : amliopidin = CCB, efek samping amliopidin itu yg muncul, maka
diganti ccb yg lain sja

57. An 50-year-old male presents to your clinic with a 5-month history of progressive shortness of breath, dyspnea on
exertion, orthopnea, and lower extremity edema. He notes “chest pressure” while walking up that is relieved with rest. On
exam, JVP:R+4 at supine position. There is an audible grade IV systolic ejection murmur heard at the base with radiation to
the carotid arteries. An EKG shows evidence of LVH. Echocardiogram shows a stenotic, calcified aortic valve with an
estimated valve area of 0.7 cm2. EF ± 40%.
Question: What management step should you recommend to improve long-term survival?
A. Surgical valvulotomy.
B. ACE inhibitor+HCTZ
C. Digoxin +Furosemide
D. Balloon valvuloplasty
E. Aortic valve replacement

58. A 55-year-old male presents with severe substernal chest pain for the last hour. It began at rest and is associated with
dyspnea and nausea. The EKG shows bradycardia with Mobitz type II second-degree AV block. Chest plain film is normal.
Question: Which of the following is likely to be found in addition on EKG?
A. ST elevation in leads V1-V3 : v1 & v3 substernal. Nyeri saat istirahat : infark
B. Deep T inversion in leads V1-V3
C. ST depression in leads I and aVL
D. ST elevation in leads II, III, and aVF.
E. No other abnormalities in EKG recording

60. Seorang laki-laki 57 tahun mempunayi tekanan darah 155/90 mm Hg pada pemeriksaan rutin. Dia telah menjalani operasi
pintas koroner 4 tahun yang lalu, setelah itu dia tidak pernah lagi mengeluh nyeri dada. Pada pemeriksaan ekokardiografi
fraksi ejeksi 55%. Pemeriksaan fisis waktu istrahat normal, dan kenaikan tekanan darah dipastikan pada dua kali kunjungan.
Question: Obat hipertensi mana yang paling cocok buat penderita ini?
A. Prazosin
B. Hydralazine
C. Beta-blockers. :pernah operasi pintas koroner = pernah infark, dikasih BB
D. ACE-inhibitors
E. Calcium antagonists

1. Obat yang tepat diberikan pada penderita HOCM adalah:


33
A. Calcium channel blockers
B. Cardiac glycoside
C. ACE inhibitors
D. Beta blockers
E. Diuretic

6. Hal dibawah ini yang berhubungan dengan stenosis mitral adalah?


A. Lebih dari 60% diderita oleh perempuan
B. Terapi paling akurat adalah rekonstruksi katup
C. Biasanya ditemukan bersama-sama dengan ASD
D. Empat puluh persen disebabkan oleh endokarditis
E. Gambaran radiologis menunjukkan honey-comb appearance

9. Pada gagal jantung sistolik terjadi perubahan hemodinamik akibat?


A. Isi sekuncup bertambah dan volume diastole akhir bertambah
B. Isi sekuncup berkurang dan volume diastole akhir bertambah
C. Isi sekuncup berkurang dan volume diastole akhir berkurang
D. Isi sekuncup berkurang dan volume diastole akhir normal
E. Isi sekuncup normal dan volume diastole akhir berkurang

10. Gagal jantung diastolik paling sering disebabkan oleh?


A. Hipertensi sistemik
B. Infark miokard akut
C. Kardiomiopati dilatatif
D. Regurgitasi mitral berat
E. Stenosis aorta yang sudah berkalsifikasi

11. Pendekatan yang dilakukan pada penderita gagal jantung stadium C adalah?
A. Diuretika saja
B. Beta blockade saja
C. Diuretika + penyekat ACE
D. Diuretika + penyekat ACE + Beta blocker
E. Diuretika + penyekat ACE + Beta blockade + Digoksin

12. Salah satu efek samping spironolakton pada pengobatan gagal jantung adalah :
A. Hipokalemia
B. Gynekomasti
C. Blok jantung
D. Asma bronchial
E. Hipotensi postural

18. Salah satu tanda perifer stenosis aorta yaitu adanya pulsasi pada uvula disebut:
A. Hill sign
B. Muller sign
C. Duroziez sign
D. Corrigan pulse
E. De-Musset sign

20. Biomarker untuk mendeteksi adanya infark miokard secara dini yang kadarnya cepat naik dan cepat turun dalam darah
adalah :
A. Troponin T
34
B. Troponin I
C. CK-MB
D. SGOT
E. LDH
23. Seorang laki-laki. 58 tahun dibawa ke UGD dengan keluhan nyeri dada hebat ditengah dada dan menjalar kelengan kiri
disertai sesak napas sewaktu makan pagi selama lebih dari 3 jam. Pemberian tablet nitrat dibawah lidah tidak mempan. Pada
pemeriksaan EKG : Gelombang T terbalik di sandapan V1-V4.
Pertanyaan: Apa diagnosis kerja anda?
A. Prinzmetal angina
B. Infark miokard akut
C. Angina pektoris stabil
D. Iskemia miokard tenang
E. Angina pektoris tak stabil

26. Seorang anak lelaki, 12 tahun datang kerumah sakit untuk pemeriksaan rutin. Ditemukan adanya bising holosistolik yang
kasar, der.III/VI pada ruang interkostal IV kiri dekat sternum. Teraba adanya thrill sistolik, S2 kesan normal. Ada clubbing
fingers tapi tidak ada sianosis.
Pertanyaan: Apa diagnosis yang paling mungkin?
A. Tetralogi Fallot
B. Insufisiensi mitral
C. Defek septum interatrial
D. Duktus arteriosus persisten
E. Defek septum interventrikular

27. Seorang anak lelaki, 6 tahun tanpa keluhan tapi diketahui ada bising jantung sejak bayi. Tidak ada riwayat sianosis
maupun squatting. Pada pemeriksaan fisis, pertumbuhan badannya sesuai umurnya. Terdengar bising sistolik tipe ejeksi
der.III/VI dengan thrill halus pada ruang interkostal II dan III kiri dekat sternum. S2 pecah melebar di basal, dan komponen
pulmonik lebih keras daripada komponen aorta. Tidak ada tanda-tanda adanya gagal jantung bendungan.

Pertanyaan: Apa diagnosis yang paling mungkin?


A. Stenosis pulmonum
B. Sindoma Eisenmenger
C. Sindroma Lutembacher
D. Defek septum interatrial
E. Defek septaum interventrikular

29. Seorang wanita berumur 55 tahun datang untuk mengontrol tekanan darahnya. Dia telah memperoleh 3 jenis obat
antihipertensi yang berbeda yakni thiazide, ACEI dan beta-blocker namun tekanan darahnya masih tetap 180/105 mmHg.
Seorang staf akhirnya mendengar adanya “bruit” di daerah abdomen.
Pertanyaan: Apa anjuran pemeriksaan yang disarankan kepada penderita ini?
A. USG abdomen
B. Arteriografi renal
C. Pielografi intravena
D. Angiografi abdominal
E. Kadar klirens kreatinin

31. Selama 1 bulan sebelum masuk ke rumah sakit, seorang pasien mengeluh bila berjalan kedua kaki terasa pegal, keluhan ini
makin lama makin bertambah. Sekarang pasien merasa ibu jari kaki kanan nyeri dan kuku mulai rusak. Penderita mempunyai
riwayat perokok 2 bungkus/hari selama 10 tahun. Keluhan diatas kemungkinan besar diakibatkan proses :
A. Vasculitis
B. Diseksi arteri femoralis
C. Thrombosis vena profunda

35
D. Aterosklerosis vena saphena
E. Aterosklerosis berat arteri tibialis

34. Pada saat pemeriksaan ekokardiography didapatkan thrombus pada left ventrikel. Dokter memberikan obat warfarin 2
mg/hari dan dianjurkan makan selama 3-4 bulan. Maksud pemberikan terapi tersebut adalah mencegah terjadinya :
A. Diseksi aorta
B. Emboli arteri
C. Arterokslerosis
D. Deep vein thrombosis
E. Sindroma koroner akut

35. Setelah melakukan pemeriksaan elektrokardiography pada seorang pasien kasus hipertensi didapatkan gambaran left
ventikular hypertrophy + strain, ST depressi di Lead II,III dan aVF. Foto toraks didapatkan kardiomegali + udema paru,
elangatio aorta. Hasil USG abdomen didapatkan aneurysma aorta dan nephrolitiasis sinistra. Gambaran diatas yang didapat
yang merupakan komplikasi kasus adalah :
A. LVF + strain
B. LVF + strain dan ST depressi
C. LVF + strain, kardiomegali dan edema paru
D. LVF + strain, kardiomegali, udema paru dan aneurysma aorta
E. LVF + strain, kardiomegali, udema paru, aneurysma aorta dan nephrolitiasis

36. Seorang anak usia 7 tahun dengan nyeri abdomen dan urine yang berwarna gelap selama 10 hari. Pada pemeriksaan fisis,
terdapat lesi kulit purpura pada badan dan ekstremitas. Urin analysis memperlihatkan hematuria dan proteinuria. Hasil tes
serologic negatif untuk P-ANCAs dan C-ANCAs. Specimen biopsi kulit memperlihatkan vaskulitis necrotizing pada
pembuluh darah kulit. Specimen biopsy ginjal memperlihatkan deposit imun kompleks pada glomerulus dimana tampak
beberapa imun kompleks yang kaya akan IgA. Manakah berikut ini yang menjadi diagnosis yang paling mungkin ?
A. Henoch-schonlein purpura
B. Polyarteritis nodosa
C. Giant cell arteritis
D. Takayasu arteritis
E. Telangiectasias

38. Seorang pria umur 50 tahun memiliki riwayat angina pectoris selama bergiat. Pada pemeriksaan fisis, tekanan darah
135/75 mmHg dan denyut jantung 79/min dan irregular. Angiografi coroner memperlihatkan penyempitan 75% pada cabang
desendens anterior dari a. coronaria kiri. Sel manakah berikut ini yang menjadi target awal timbulnya lesi pada arteri coronaria
?
A. Monosit
B. Platelets
C. Neutrofil
D. Sel endotel
E. Sel otot polos

39. Suatu penelitian mengenai pembetukan ateroma yang dapat menimbulkan komplikasi atherosclerotic, mengevaluasi
faktor-faktor resiko pada satu populasi penduduk. Didapatkan 3

faktor yang berperan sebagai penyebab atherosclerosis: merokok, hipertensi dan hiperkolesterolemia. Faktor-faktor ini
kemudian dianalisis hubungannya dengan atherogenesis pada percobaan binatang. Kejadian manakah berikut ini yang timbul
dari ketiga faktor tersebut ?
A. Inhibisi oxidasi LDL
B. Jejas endotel dan sequelaenya
C. Perubahan reseptor lipoprotein hepatic
D. Perubahan sel otot polos menjadi foam cell
36
E. Perubahan faktor-faktor endogen yang mengatur tonus vasomotor

41. Seorang laki-laki, 72 tahun, masuk ke rumah sakit karena nyeri dada yang menyebar ke tangan kiri. Hasil pemeriksaan
EKG memperlihatkan terjadinya infark miokard pada daerah dinding posterior ventrikel kiri. Serabut saraf manakah berikut
ini yang berperan dalam penyebaran nyeri ke lengan kiri penderita tersebut?
A. N. Vagus
B. N. Phrenicus
C. N. Suprascapularis
D. N. Intercostobrachialis
E. N. Splanchnicus major

42. Seorang laki-laki, 69 tahun, masuk ke rumah sakit dengan nyeri dada. Hasil pemeriksaan EKG menunjukkan hypokinetic
ventricular septal muscle, infark miokard pada 2/3 anterior septum interventricularis dan dinding anterior ventrikel kiri. Juga
terlihat left bundle branch block. Arteri mana berikut ini yang mengalami oklusi?
A. A.circumflexus
B. A. coronarius dextra proximal
C. A. coronarius sinistra proximal
D. A. interventricularis anterior sinistra proximal
E. A. interventricularis posterior

44. Seorang pria parubaya setelah suatu episode rasa tertekan retrosternal dengan radiasi ke leher disertai mual dan diaphoresis
saat istirahat, didiagnosa unstable angina dengan kemungkinan infark miokard. Kemudian pemeriksaan laboratorium dijumpai
penungkatan kadar serum troponin I dan enzim-enzim jantung yang menetapkan diagnosis. Jika penderita diatas sesungguhnya
menderita angina, perubahan metabolism yang mana berikut ini dijumpai pada daerah yang terkena?
A. Meningkatnya pembentukan laktat

B. Meningkatnya fosforilasi okasidatif


C. Meningkatnya laju oksidasi asam lemak
D. Meningkatnya pembentukan benda keton
E. Meningkatnya perubahan piruvat ke asetil-KoA

45. Jika penderita diatas sesungguhnya menderita angina sebagai lesi fokal dimana hanya satu bagian otot jantung yang
terkena sementara bagian lain tetap menerima pasokan oksigen yang cukup memungkinkan berfungsinya suatu biochemical
shuttle (gerakan ulang alik) yaitu :
A. Glycerol 3-phosphate shuttle
B. Creatine Phosphate shuttle
C. Malate aspartate shuttle
D. Carnitine shuttle
E. Citrate shuttle

46. Yang mana mekanisme transpor elektron melintasi barrier mitokondria sehubungan dengan shuttle diatas?
A. Glutamate-α-ketoglutarate antiporter
B. Aspartate-α-ketoglutarate symporter
C. Malate-α-ketoglutarate antiporter
D. Malate-glutamate symporter
E. Malate-aspartate antiporter

47. Seorang pasien pria usia 67 tahun mengeluh sakit dada seperti tertarik sampai punggu dibelakang, dialami 1 hari lalu,
perasaan lemah badan. Pasien mempunyai riwayat perokok 1-2 bungkus/hari dan hipertensi lama tanpa control yang baik.
Pada pemeriksaan Fisis TD 170/80 mmHg, Denyut jantung 98 x/mnt, EKG normal sinus ritme foto toraks dalam batas normal.
Dokter membuat diagnose sementara suatu PJK dan Hipertensi serta membuat diagnose banding suatu :
37
A. Perikarditis
B. Diseksi aorta
C. Pericard effusion
D. Sick sinus sindrome
E. Kardiomiopati dilatasi

48. Seorang perempuan usia 24 tahun mengeluh pusing sejak 1 minggu terakhir, terus menerus. Pada pemeriksaan fisis
didapatkan tekanan darah 200/110 mmHg, denyut jantung 90 x/mnt. Riwayat hipertensi sebelumnya sekitar 6 bulan dan telah
mendapatkan obat captopril 3 x 50 mg/hari dan makan rutin. Dokter mencurigai suatu kasus :
A. Aneurysma aorta
B. Hipertensi primer
C. Hipertension heart disease
D. Hipertensi kausa RAA sistem
E. Hipertensi kausa stenosis a.renalis

49. Seorang pasien pria usia 65 tahun, mengeluh sakit kepala, sesak napas, betis kanan bengkak, ibu jari kaki kiri kehitaman
disertai sendi lutut kaki kanan nyeri bila digerakkan. Penderita mempunyai riwayat diabetes mellitus dan perokok lama.
Keluhan pasien yang manakah yang dokter akan mendiagnosa banding dengan deep vein thrombosis :
A. Sakit kepala
B. Sesak napas
C. Betis kanan bengkak
D. Ibu jari kaki kiri kehitaman
E. Sendi lutut kaki kanan nyeri bila digerakkan

50. Seminggu setelah dilakukan operasi tulang panggul, penderita mengalami pembengkakan pada ekstremitas inferior sinistra
mulai femur ke distal, tidak nyeri dan warna sama dengan kulit sekitar, fluktuasi (-), demam (-). Tiba-tiba pasien mengeluh
sesak napas tiba-tiba, denyut jantung 110 x/mnt, ronki (-), EKG : dalam batas normal. Diagnosa sementara pasien :
A. Pneumoni
B. Emboli Paru

C. Udem Paru Akut


D. Congestive Heart Failure
E. Acute Coronary Syndrome

51. Selama 1 bulan sebelum masuk ke rumah sakit, seorang pasien mengeluh bila berjalan kedua kaki terasa pegal, keluhan ini
makin lama makin bertambah. Sekarang pasien merasa ibu jari kaki kanan nyeri dan kuku mulai rusak. Penderita mempunyai
riwayat perokok 2 bungkus/hari selama 10 tahun. Keluhan diatas kemungkinan besar diakibatkan proses :
A. Vasculitis
B. Diseksi arteri femoralis
C. Thrombosis vena profunda
D. Aterosklerosis vena saphena
E. Aterosklerosis berat arteri tibialis

52. Seorang lelaki 55 tahun datang ke poliklinik dengan keluhan sesak napas berat sejak beberapa jam yang lalu disertai batuk
berdahak. Tidak ada faktor-faktor risiko koroner yang disandang. Ada riwayat trauma tajam pada bagian pinggang bawah
kanan 2 tahun lalu. Pada pemeriksaan fisis: ronki basah kasar dikedua basal paru. Tungkai bawah kanan hangat dan bengkak.
Terdengar bising kontinyu pada daerah arteri femoralis. Penderita didiagnosis sebagai gagal jantung.
Pertanyaan: Apa jenis gagal jantung yang diderita oleh lelaki ini?
A. Gagal jantung sistolik
B. Gagal jantung high output
C. Gagal ventrikel kiri backward
D. Gagal jantung bendungan akut

38
E. Gagal ventrikel kiri kronis eksaserbasi akut

53. Seorang penderita laki-laki 73 tahun di deteksi adanya gangguan irama berupa pulsus irregular di poliklinik. Tidak ada
keluhan baik waktu istrahat maupun waktu bergiat. Pada pemeriksaan EKG: Gelombang P tidak terlihat, interval RR tidak
teratur dengan laju 74/min. Gambaran EKG 3 tahun lalu masih irama sinus normal.

Pertanyaan: Obat apa yang anda sarankan untuk dikonsumsi secara rutin pada penderita ini untuk mencegah timbulnya
komplikasi tromboemboli?
A. Digoxin + aspirin
B. Klopidogrel+ aspirin
C. Amiodarone + digoxin
D. Aspirin + antikoagulan
E. Antikoagulan+ klopidogrel

54. Pada kasus kaki diabetic seorang dokter memberikan obat ceftriaxon 1 gr/12 jam/hari, insulin sesuai kadar glukosa darah,
aspilet 80 mg/hari, cilostazol 2 x 50 mg/hari dan vitamin B1, B6 dan B12 1 tablet/hari serta captopril 2 x 50 mg/hari. Karena
tidak ada perbaikan maka dilakukan angiography didapatkan stenosis pada arteri tibialis anterior. Selanjutkan dianjurkan
tindakan/pemberian :
A. Bypass
B. Amputasi
C. Clopidogrel
D. PTCA + stent
E. Thromboektomi

58. Seorang pria umur 25 tahun meninggal secara tiba-tiba. Pada autopsi, jantungnya membesar yang diakibatkan dilatasi
ventrikel kanan. Ventrikel kiri normal. Tidak ada atherosclerosis coronaria. Potongan melintang jantung memperlihatkan
ventrikel kanan menipis dan gambaran mikroskopis meliputi infiltrasi lemak miokardium dan fibrosis, tidak ada tanda-tanda
inflamasi. Manakah berikut ini yang menyebabkan kematian pada pria tersebut.
A. Hipertensi
B. Kardiomiopati
C. Chagas’ disease
D. Post terapi radiasi
E. Long QT syndrome

59. Wanita umur 48 tahun mengalami sesak napas dalam 2 hari terakhir. Ia mengalami cardiac arrest dan tidak dapat
diselamatkan. Pada autopsy, gambaran mikroskopik jantung memperlihatkan nekrosis koagulatif. Manakah berikut ini yang
menjadi diagnose dari wanita tersebut ?
A. Miokarditis rematik akut
B. Kardiomiopati restriktif
C. Miokarditis virus
D. Infark miokard
E. Emboli septik

60. Seorang penderita TBC kronis, datang ke rumah sakit dengan keluhan sesak napas sejak 1 bulan terakhir, makin
memberat. Tidak ada nyeri dada. Fisis jantung : kesan batas jantung kiri dan kanan melebar. Amplitudo nadi radialis berubah-
ubah sesuai dengan inspirasi dan ekspirasi. Dianjurkan pem.ekokardiografi.
Pertanyaan. Maksud pemeriksaan ekokardiografi untuk menemukan adanya :
A. Stenosis Mitral
B. Defek septum atrial
C. Tamponade jantung
D. Endokarditis bakterialis
E. Hipertrofi ventrikel kanan

39
61. Wanita 60 tahun mengeluh sesak napas. Hal ini sudah lama dialami dan sudah berobat lama tapi tidak teratur. Pem.fisis:
TD: 180/100 mmHg, Nadi 120/mnt, ronki basal paru bilateral, udem tungkai bilateral. Foto toraks: kardiomegali.

Pertanyaan:Gejala akut mana yang timbul akibat mekanisme kompensasi jantung?


A. Kardiomegali
B. Nadi 120 x/mnt

C. Ronki paru bilateral


D. Udema tungkai bilateral
E. Tekanan darah 180/100 mmHg

63. A 50 year-old man presents to cardiovascular care unit with shortness of breath. Past medical history is significant only for
chest pain on exertion. Echocardiogram reveals LV hypokinesis with an LVEF of 30%. He receives appropriate diuresis and
therapy with ACEI and digoxin.
Question: What additional work up would be appropriate?
A. Chest X-ray
B. Exercise stress test
C. Cardiac catheterization
D. Holter’s ECG monitoring
E. Level of brain natriuretic peptide

66. A 60 year-old woman presents to emergency room with complaining of chest pain. Her BP is 75/50 mmHg, her radial
pulse is faint and slow 50/min. She is diaphoretic, her lips are dusky. JVP is R+5 cm at 45 degree, but the lung fields are clear.
In the ECG you find ST elevation in leads II, III, and aVF.
Question: What’s going on in this patient?
A. Acute inferior myocardial infarction with hypotension
B. Acute inferior myocardial infarction with RV infarction
C. Acute inferior myocardial infarction with cardiogenic shock
D. Acute inferior myocardial infarction with hypotension and RV failure
E. Acute inferior myocardial infarction with RV infarction and RV failure

68. An 70 year-old woman with a history of pulmonary edema once again presents in heart failure. Past admissions has noted
a loud, harsh systolic murmur at the right ICS-2 radiating to the neck. Now, you note the murmur, but it is barely audible on
your exam.
Question: What cardiovascular lesion should you suspect?
A. Occult aortic stenosis
B. Arterio-venous fistula
C. Adult pulmonal stenosis
D. Assymetrical septal hypertrophy
E. Idiopathic hypertrophic subaortic stenosis

69. Sustained ventricular tachycardia is well controlled in a 55 year-old man post non-Q wave myocardial infarction with
lidocaine 2 microgram/ml. He also received isosorbid dinitrate and diltiazem. He has a known history of ischemic
cardiomyopathy. Day 3 of his admission, his speech becomes slurred, he is lethargic, and, when aroused, becomes very
agitated.

Question: What should you do?


A. You add alprazolam as tranquilizers
B. Stop the administration of lidocaine
C. Change lidocaine with amiodarone
D. Perform DC-shock 360 Joule
E. Give the patient NaCl 3%
40
70. A 40 year-old man comes to the hospital with complaining of chest pain, fatigue, and breathlessness. ECG recording
reveals atrial fibrillation with HR about 110/min. Physical exam: a forceful apical impulse, S4, ejection systolic murmur
beginning in mid-systole. Echocardiographic examination reveals systolic anterior movement of the mitral valve. Your
diagnosis is hypertrophic cardiomyopathy.

Question: Why does this patient complain of chest pain?


A. Rapid atrial fibrillation
B. Increased cardiac work
C. Impaired diastolic filling
D. Decreased cardiac output
E. Increased outflow tract obstruction

71. A 59 year-old woman presents for the first time with untreated congestive heart failure. Physical exam: BP 110/80 mmHg,
HR : 104/min, RR: 28/min. There are pitting edema in the extremities. You ask urinalysis and urine biochemistry besides
routine blood analysis.
Question: Which of the following is the most likely showed by urinalysis and urine biochemistry?
A. Increased urinary chloride content
B. Decreased urinary sodium content
C. Low urine specific gravity
D. Red blood cell casts
E. Proteinuria

72. A 72 year-old man is found to have an irregular pulse rate on a routine visit. He is experiencing no new symptoms at rest
or on exertion. The ECG recordings reveal invisible P wave, an irregular RR interval at a rate 74/min. On his previous ECG
from 3 years ago he was in sinus rhythm.

Question: Which of the following is the most appropriate next step in management?
A. Thrombolytic therapy
B. Acetyl salicylic acid
C. Cardioversion

D. Anticoagulant
E. Amiodarone

74. A 60 year-old woman presents to the hospital with complain of palpitations, anxiety, diarrhea and weight loss. On
examination the patient is underweight, her BP is 160/60 mmHg, HR 120-140/min, RR 28/min, the extremities are clammy.
Question: Which of the following is the most likely cardiac finding?
A. Paroxysmal atrial fibrillation
B. Arterio-venous fistula
C. Aortic insufficiency
D. Pericardial effusion
E. Atrial flutter

75. A 22 year-old woman complains of palpitations and has regular heartbeat at a rate of 170/min, with a BP of 110/70 mmHg.
The rate abruptly changes to 75/min after applying carotid sinus pressure.

Question: Which of the following is the most likely diagnosis?


A. Sinus tachycardia
B. Paroxysmal atrial flutter
C. Paroysmal atrial fibrillation
D. Paroxysmal ventricular tachycardia

41
E. Paroxysmal supraventricular tachycardia

76. Auscultation of the heart of a 15 year-old boy reveals an increased intensity of the pulmonary component of S-2. He
complains of dyspnea on exertion but no other cardiac or pulmonary symptoms.

Question: Which of the following explanations is the most likely cause of dyspnea?
A. Aortic stenosis
B. Pulmonary stenosis
C. Mitral regurgitation
D. Systemic hypertension
E. Pulmonary hypertension

77. A 60 year-old man comes to the emergency room with new-onset syncope. On examination, his BP of 110/95 mmHg and a
harsh systolic ejection murmur at the right ICS-2, radiating to both carotids.

Question: Auscultation of the S-2 at the base might reveal which of the following?
A. It is diminished
B. It is widely split
C. It is accentuated
D. It shows fixed splitting
E. It is normal in character

78. A 20 year old-man presents to the hospital with complain of breathlessness, fatigue and productive coughing. On exam, his
BP 120/75 mmHg, HR 90/min, RR 28/min. There is a systolic murmur grade III/VI in the left ICS-2. On the CXR the
pulmonary artery and its branches prominent.

Question: Auscultation of the S-2 at the base might reveal which of the following?
A. It is diminished
B. It is accentuated

C. It is widely split
D. It shows fixed splitting
E. It is normal in character

79. A 72 year-old woman has ne onset supraventricular tachycardia with a rate 150/min. She is hemodynamically stable with a
BP of 155/90 mmHg. Now she is experiencing palpitations and a little bit headache. There is no history regarding drug
hypersensitivity.
Question: Which of the following drugs is the best intravenous choice for controlling the heart rate?
A. Diltiazem
B. Amiodarone
C. Aminophyllin
D. Sulfas chinidine
E. Magnesium sulfat

80. A 65 year-old man complains of postural hypotension with dizziness. His BP is 110/80 mmHg supine and 85/70 mmHg
standing after 2 minutes, the pulse rate remains the same at 80/min.
Question: Which of the following is the most likely diagnosis?
A. Possibly DM
B. Hypothyroidism
C. Diuretic therapy
D. Amlodipine therapy
E. Essential hypotension

42
81. A 70 year-old man presents to the hospital with complain of fatigue and dyspnea on exertion. His BP is 165/60 mmH, and
pulse 75/min. On auscultation you hear the S-2 is diminished and there is an decrescendo diastolic murmur that radiates from
right ICS-3 to the apex cordis.

Question: Which of the following arterial pulse finding is the most appropriate in this patient?
A. Dicrotic pulse
B. Pulsus bigeminus
C. Pulsus paradoxus
D. Hyperkinetic pulse
E. Delayed femoral pulse

82. A 50 year-old man presents to emergency department with complain of substernal chest pain. His chest pain usually occurs
during physical activity. The exercise electrocardiogram is positive but he has a normal coronary angiogram.

Question: What is your diagnosis?


A. Syndrome X
B. Silent ischemia
C. Angina decubitus
D. Prinzmetal angina
E. Barlow’s syndrome

83. Three months after an anterior MI, a 73-year-old man has a follow-up ECG. He is clinically feeling well with no further
angina symptoms. His ECG shows Q waves in anterior leads with persistent ST-segment elevation.

Question: The current ECG is most compatible with which of the following diagnosis ?
A. Acute infarction
B. Early repolarization
C. Ventricular aneurysm
D. Hibernating myocardium
E. Silent myocardial infarction

84. A 45 year-old woman with new-onset aortic regurgitation is found to have aortic dissection of the ascending aorta and
aortic arch by echocardiography. She is relatively asymptomatic.
Question: Which of the following is the best management?
A. Angioplasty
B. Surgical correction
C. Oral warfarin therapy

D. Oral atenolol therapy with monitoring


E. Intravenous isosorbid dinitrate therapy

85. A 50 year-old man with hypertension, diabetes, and hyperlipidemia comes to the outpatient clinic complaining of pain in
his calves whwn he walks 200-300 meters.
Question: What therapy might offer him the greatest benefit in symptom reduction and in overall mortality?
A. Aspirin therapy
B. Cilostasol therapy
C. Smoking cessation
D. Lipid lowering agent
E. Limb revascularization procedure

89. A 45 year-old smoker presents with symptoms of palpitations and is found to be in a regular wide complex tachycardia at a
rate of 170 bpm. He is reclining comfortably on a stretcher while you interview him.
Question: What can you safely assume about the etiology of his tachycardia?
43
A. Nodal tachycardia
B. Torsade de pointes
C. Ventricular tachycardia
D. Supraventricular tachycardia
E. Coarse ventricular fibrillation

91. A 33 year-old female comes to you for a physical and you notice a harsh systolic murmur at the apex that is also heard at
the base. The murmur increases on standing and Valsalva and decreases with handgrip.
Question: What is the most likely finding on echocardiography?
A. Mitral valve prolapsed
B. Severe mitral regurgitation
C. Assymetrical septal hypertrophy
D. Aortic stenosis with mitral regurgitation
E. Idiopathic hypertrophic subaortic stenosis

92. Two weeks ago you doubled the diuretic dose of an 80 year-old woman receiving standard therapy for systolic heart
failure. She is subsequently admitted to cardiovascular care unit with confusion, nausea and vomiting and third degree AV
block in EKG.
Question: What the most important thing should you check?
A. The level of PaCO2
B. Serum digoxin level
C. Chloride concentration
D. The level of magnesium
E. The level of sodium bicarbonate

93. A 48 year-old man is admitted to the hospital for chest pain since for the past two days. On examination, Bp 110/80
mmHg, HR 110 bpm, RR 26/min, T 38.5 0 C. EKG reveals ST segment elevation in limbs leads as well as V2-V6. He has end
stage renal disease as a consequence of diabetes.

Question: Which of the following is usually find on physical examination?


A. Pericardial friction rub
B. Pericardial knock
C. Paradoxal pulse
D. Kusmaull’s sign
E. Gallop rhythm

94. A 65 year-old woman develops exertional angina and has two episodes of syncope. Examination shows BP 110/90 mmHg,
HR 100/min. There is a sign of anacrotic pulse, and on auscultation you can hear a soft second heart sound and murmur at the
basis.

Question: What kind of murmur can be heard at the basis of the heart of this patient?
A. A decrescendo diastolic murmur
B. A harsh holosystolic murmur
C. A systolic ejection murmur
D. A proto-diastolic murmur
E. A late systolic murmur

95. A 25 year-old man complains of left precordial chest pain that radiates to the left shoulder but not down the left arm. The
pain is accentuated by inspiration and relieved by sitting up. The pain is accompanied by fever and chills. His BP is 105/75
mmHg, pulse 110/min and regular, temperature 380 C. On the third hospital day, the patient’s blood pressure is 70/50 mmHg,
JVP R+5 cm and he goes into CHF.

Question: Which of the following is the most likely diagnosis on the third hospital day?
44
A. Pulmonary emboli
B. Cardiac tamponade
C. Pericardial inflammation
D. Rupture of the chorda tendineae
E. Extension of a myocardial infarction

97. A 65 year-old female presents to the hospital with sudden chest pain and shortness of breath. His initial ECG reveals 2 mm
ST depression in lead V1-V4 with inverted T waves. On auscultation she has bibasilar rales and CXR reveals moderate
pulmonary edema. Serial ECG’s and CPK’s confirm a NSTEMI. With furosemide, her pulmonary edema resolves within 24
hours.
Question: What is the most appropriate management strategy at this point?
A. Cardiac catheterization with coronary angiography
B. Cardiac catheterization and ventriculography
C. Dobutamine stress electrocardiography
D. Multisliced compute-tomography
E. Trans-thoracal echocardiography

98. A 50 year-old man comes to the hospital with fever, chest pain and a sign of pleuropericarditis. Two weeks ago he had
post acute myocardial infarction. On exam BP is 110/80 mmHg, HR is 110 bpm and RR is 28/min. CXR reveals a pleural
effusion.

Question: What is your diagnosis?


A. Acute viral pericarditis
B. Reinfarction with pneumonia
C. Non-STEMI with pleuropericarditis
D. Post myocardial infarction syndrome
E. Unstable angina pectoris with pleuritis

99. A 70 year-old male is scheduled for elective abdominal aortic aneurysm repair. On the evening prior to this patient’s
surgery, his ECG shows occasional PVC. There are no symptoms at all. The anaesthesiologist is concerned that the patient
may go into ventricular tachycardia (VT) and asks for a cardiology consult.

Question: What do you recommend?


A. Administer amiodarone
B. Doesn’t require treatment
C. Administer chloride potassium
D. Perform 24 hours’ ECG monitoring
E. Use temporary pacemaker for prevention of VT

100. A 40 year-old man comes to the hospital with complaining of central chest pain, stabbing sensation with radiation to
shoulders and upper arm since yesterday. It is relieved by sitting up and leaning forward, and aggravated by lying down. His
BP is 100/80 mmHg, HR is 105 bpm, temperature is 38.5 0 C.

Question: Before you make a definite diagnosis, what kind of medication do you recommend to relieve the symptoms
immediately?
A. Streptokinase
B. Morphine sulfat
C. Isosorbid dinitrate
D. Acetyl salicylic acid
E. Methyl prednisolone

5. Pemberian HCT maupun furosemide jangka panjang pada penderita ini dapat menimbulkan penyulit berupa hal-hal di
bawah ini, KECUALI :
45
A. Hypocalcaemia
B. Hyperuricemia
C. Hypokalemia
D. Hypomagnesaemia
E. Hyponatremia

14. Gambaran radiologist yang khas menunjang diagnosis stenosis mitral adalah :
A. Kardiomegali
B. Double contour
C. Cardiac waist menghilang
D. A + B benar
E. A + C benar

15. Tindakan yang bersifat “kardiologi intervensi” pada penderita ms adalah :


A. Commisurotomy
B. Ballon mitral valve

C. Valve reconstruction
D. A + B benar
E. A + C benar
Three days prior admission, 1 60 years-old man complained mild to moderate sub sterna heaviness for 29 minutes whilst
resting; numbness in left arm. In the morning of admission he awakened by recurrence of symptoms but more severe lasting
30 minutes, accompanied by choking sensation. Background : obese patient, sedentary life, smoked, 1,5 packs/day for 15
years, jugular venous pressure 5 cm at 45 degrees, normal CPK, slight elevated CKMB, cardiac troponin 1 elevated,
leucocytosis, hypercholesterolemia, systolic ejection murmur grade 2/6. EKG : ST segment depression 2-3 min with inverted,
T wave in leads I,aIV an 11,111,aVI. Radiologist : accentuation of pulmonary vascular markings.
16. Diangosis yang memungkinkan untuk penderita di atas adalah :
A. Infark miokard transmural (dengan Q wave patolops)
B. Infark miokard nontransmural (non-Q wave)
C. Angina pectoris tidak stabil
D. Angina pectoris kronis
E. Angina prinzmetal

18. Menilai keadaan penderita di atas baik klinis maupun laboratorium, maka penderita ini tergolong penderita dengan :
A. Resiko rendah
B. Resiko rendah–sedang
C. Resiko sedang
D. Resiko sedang-tinggi
E. Resiko tinggi

20. Bila dengan terapi di atas ditambah dengan terapi lainnya seperti beta-blocker, ACE inhibitor, antiplatelet, tranquilizer
penderita masih mengalami nyeri dada, maka dianjurkan segera melakukan pemeriksaan :
A. Cxghest X-Ray
B. Echokardiografi
C. Exercise ECG test
D. Angiografi koroner
E. Semua benar

24. Which statement is correct for aortic stenosis :


A. The classical symptoms are angina pectoris, syncope or pre-syncope, and heart failure.
B. In symptomatic patients, the 5 years survival rate is < 50 %
C. Once syncope and failure occurs, the average survival is only 1,5 years
46
D. This disease is more frequent in males
E. All correct

27. DRESSLER’S syndrome adalah :


A. Endokarditis pada penyakit jantung congenital
B. Perikarditis pasca infark miokard
C. Miokarditis akibat uremi
D. Tamponade cordis akibat TBC
E. Semua benar

38. Seorang wanita muda 25 tahun masuk rumah sakit dengan keluhan palpitasi. Riwayat sebelumnya sehat, setiap hari olah
raga. Pada EKG tampak supraventikular takikardi dengan laju jantung 180 x/min. Tekanan darah 110/70 mmHg. Obat yang
segera diberikan adalah :
A. Nifedipin
B. Furosemide
C. Verapamil
D. Kaptopril
E. Disopiramide

43. Bila penderita tersebut terbukti juga menderita penyakit jantung koroner, maka anti-hypertensive yang menjadi pilihan
utama adalah :
A. Diuretik
B. Antagonis kalsium
C. Simpatolitik sentral
D. Semua bisa menjadi pilihan utama
E. Tidak ada dari ketiga obat tersebut di atas
44. Terdapat pada serta purkinje, KECUALI :
A. Myofibril
B. Butir glycogen
C. Sarcolemma
D. Sel basket
E. Karyolemma
45. Mixed type (hybrid type) dari pembuluh darah ialah :
A. Precapiler
B. Arteri axillaris
C. Aorta
D. arteri brachialis
E. Vena besar
46. Beberapa fungsi sistem kardiovaskuler yang bertujuan untuk mempertahankan proses homeostasis adalah sebagai berikut,
KECUALI :
A. Penyediaan berbagai bahan nutrisi dan oksigen bagi sel dan jaringan
B. Mepertahankan hasil metabolisme
C. Transport berbagai bahan dari satu organ ke organ lainnya
D. Mengangkut berbagai antibodi yang diperlukan untuk pertahanan tubuh
E. Mengatur konsentrasi air dan ion pada sel dan jaringan
47. Perluasan potensial aksi dari sumber impuls yaitu sel-sel pacemaker nodus SA menuju ke nodus AV melalui :
A. Traktus intermodal
B. Berkas His
C. Serabut purkinje
D. Serabut otot ventrikel
E. Traktus internodal & serabyuut otot atria

47
1. Seorang wanita 25 tahun datang ke klinik untuk “medical check up” karena adanya keluhan cepat lelah dan berdebar-debar
akhir-akhir ini, pemfis keadaan umum baik, TD 120/80 mmHg, nadi 90/menit. BJ I normal, BJ II “wide fixed split”, EKG
menunjukkan irama sinus dengan tanda-tanda RVH.
Pertanyaan : Apa diagnosis sementara kasus di atas?
A. MR
B. TOF
C. ASD
D. VSD
E. MVP (mitral valve prolaps)
2. Pemeriksaan penunjang non-invasif yang dianjurkan untuk menegakkan dianosis?

A. CXR-PA
B. CT-scanning
C. Echocardighraphy
D. Thalium scintigraphy
E. MRI

3. Gambaran radiologist yang terlihat pada kasus di atas adalah?

A. Kardiomegali dengan densitas ganda (LAH)


B. Kardiomegali dengan penonjolan aurikel kiri(LAH)
C. Kardiomegali dengan penyempitan ruang retrosternal

D. Kardiomegali dengan penyempitan ruang retrokardial(LVH)


E. Kardiomegali dengan penekanan cabang bronkus kiri(LAH)

4. Pembesaran jantung pada kasus di atas terutama disebabkan oleh?


A. RA volume overload
B. RV volume overload
C. LV volume overload
D. LV pressure overload
E. RV pressure overload
5. Kelainan bunyi jantung di atas

48

You might also like